exam 7 review

Pataasin ang iyong marka sa homework at exams ngayon gamit ang Quizwiz!

what does high creatinine indicate?

renal disease

creatinine indications

renal function

Using this table above, determine the p-value of the relationship between transmission rates and temperature. Then, decide whether the transmission of virus and temperature are related based on this value. A. P value: 0.690, not related B. P value: 0.857, related C. P value: 0.690, related D. P value: 0.857, not related E. Need more data to determine p value's significance

D. P value: 0.857, not related

A 65-year-old man came back from Malaysia celebrating he and his wife's retirement. While visiting they went on a jungle safari. Upon boarding the plane, he had a fever, headache, abdominal pain, and a productive cough. Thinking nothing of it, he went home to get some rest. The next day, he had another fever accompanied by chills. His wife took him to the urgent care and the attending physician ordered a blood test. Upon viewing the blood smear under a light microscope, fine ring forms and double chromatin dots were present. Although this parasite is difficult to detect on its own, which parasite would you most likely treat it as? A. P. vivax B. P. ovale C. P. malariae D. P. falciparum E. T. brucei

D. P. falciparum

A parasitologist visited a clinic in Nigeria to help eliminate some of the factors causing local malaria infections. He was notified one day that a 3 year-old died from severe kidney damage, just a week after being bit by a mosquito. Which Plasmodium species most likely caused the infection and subsequent death of the child? A. P. vivax B. P. ovale C. P. malariae D. P. falciparum E. P. knowlesi

D. P. falciparum

A 26-year-old male was exploring an old sewer when he discovered a pack of rats feeding on some garbage that had been left behind. He decided to pick up one of the rats but was unsuccessful as they ran away. As he was walking out of the sewer, he noticed that he had gotten a flea bite while in there. He did not think anything of it and continued on with his day. 5 days later, he went to the hospital with a high fever and painful bubo in his groin area. The patient was not treated quickly enough and developed bacteremia and later died. What is the pathogen that killed this patient? A. Yersinia enterocolitica B. Yersinia pestis C. Klebsiella granulomatis D. Salmonella typhi E. Yersinia pseudotuberculosis

B. Yersinia pestis

A 35-year-old male chef initially presents to the clinic with nausea, vomiting, and anorexia. Upon further examination, the patient has a fever and complains of headache, myalgia, and malaise. The patient also mentions he recently traveled to Cambodia. The clinic cultured the organism on MacConkey agar and noticed the colonies grew colorless and were resistant to bile salts. The patient was educated about chronic colonization of the gallbladder and prescribed antibiotics. What is the organism responsible for causing the patient's symptoms? A. Salmonella Typhi B. Escherichia coli C. Klebsiella pneumoniae D. Salmonella Choleraesuis E. Citrobacter koseri

A. Salmonella Typhi

A 23 year old female visits the local Urgent Care because of recent pain and swelling in her axillary region. The patient informs the physician of her high fever that preceded the painful bubo. The physician asks the patient about the scratch on her arm, which she explains was the result of trying to put the stray cat she adopted almost a week ago in the carrier for its veterinarian appointment for a flea bath. What virulence gene allows this bacteria to prevent opsonization and phagocytic migration, and spread rapidly throughout the host? A. virB operon B. Fraction 1 gene C. pInv genes D. Plasminogen activator protease gene E. Locus of enterocyte effacement pathogenicity island

D. Plasminogen activator protease gene

What does high C- peptide indicate?

- Type 2 diabetes (normal to high - Insulinoma - Renal failure

Glucose tolerance indications

- Typically is used to diagnose diabetes - look for hypoglycemia

What does high indirect (unconjugated) bilirubin indicate?

- accelerated erythrocyte (RBC) hemolysis - hepatitis - drugs

Anion gap indications

- acid/base disorders - difference between the cations and anions in the extracellular space

What does high aspartate aminotransferase (AST) indicate?

- liver disease - skeletal muscle disease - acute hemolytic anemia - acute pancreatitis

What does low potassium indicate?

- deficient intake - alkalosis - burns - diuretics - increase in insulin or glucose HYPOKALEMIA

Aspartate aminotransferase (AST) indications

- evaluation of patients with suspected hepatocellular diseases - Serum AST levels become elevated 8 hours after cell injury, peak at 24 to 36 hours, and return to normal in 3 to 7 days. - If the cellular injury is chronic, levels will be persistently elevated.

What does low anion gap indicate?

- excess alkali ingestion - chronic vomiting - multiple myeloma

What does low LDL or VLDL indicate?

- familial hypolipoproteinemia - hyperthyroidism

What does high anion gap indicate?

- lactic (metabolic) acidosis - ketoacidosis

What is the correct order for WBC counts? (most to least)

1. neutrophils 2. lymphocytes 3. monocytes 4. eosinophils 5. basophils

A 4-year-old male presents to the clinic with symptoms of abdominal pain, cough, seizures, and pruritic skin lesions. Upon physical exam, the child is observed to have hepatosplenomegaly. The patient and his family recently moved to the area and he was enrolled in preschool, and his parents state that the patient has especially loved the petting zoo they have at the school, consisting of dogs, chickens, and cats that live on the property. Following diagnostic blood tests, the patient is found to have elevated eosinophil levels. How might the patient have contracted this nematode? A) Playing with infected animals at school B) Contaminated clothing/toys at the school C) Inhaling the infected egg-laden dust in the family's new home D) Playing outside with bare feet E) Eating a piece of infected bacon

A) Playing with infected animals at school

Mariah decides to celebrate her 21st birthday in Malaysia where she and her 6 best friends attend a cookout in the community where they ate roast pork, mango margaritas and/or a local vegetable dish. When she returned home 6 hours later, she had diffuse abdominal pain, watery nonbloody diarrhea, nausea, and vomiting. Two of her other friends also had similar symptoms. Her vegetarian friend did not get sick at all and her remaining friends did not show any symptoms. Mariah and her sick friends go to the doctor who suspects a parasitic infection. This is confirmed by the results of a wet mount stool specimen. All of the collected stool specimen had the same pathogen and only contained sporocysts. What pathogen is responsible for Mariah and her friends getting sick and what is most likely the source of the pathogen? A) Sarcocystis spp.; infected meat B) Cystisopora spp.; mango margaritas C) Heliobacter pylori; local vegetable dish D) COVID19; mango margaritas E) HPV; infected meat

A) Sarcocystis spp.; infected meat

A 17-year-old teenager presents to the clinic with complaints of watery diarrhea, vomiting and stomach pain. He reports being at a party about 6 hours earlier that day where food was served. The meal included fried rice, meat and vegetables that seemed rather raw. After 32 hours the patient's health seemed to be back to normal. Given the signs and symptoms displayed by the patient, which of the following organisms most likely caused his illness? A. Bacillus cereus B. E. coli C. Salmonella typhi D. Vibrio parahaemolyticus E. Norovirus

A. Bacillus cereus

Which values match the reproductive ratio and serial interval of COVID-19, respectively? A. 2.28 and 3.77 B. 2.28 and 1.95 C. 3.62 and 3.77 D. 1.86 and 3.28 E. 2.98 and 3.53

A. 2.28 and 3.77

The longest infectious period detected in Viral Culture for someone with severe symptoms for SARS-CoV2 in "The Duration of infectiousness of individuals infected with SARS-CoV2" of The Argument Presentation was: A. 32 days B. 20 days C. 13 days D. 10 days

A. 32 days

23-year-old female presents to her PCP with complaints of pelvic pain, increased urge to urinate, pain with urination, and blood in the urine for 2 days. She reports no recent travel or GI symptoms. Urine culture and lab tests reveal lactose positive, indole positive, gram negative rods. She is diagnosed with a bacterial UTI and treated with antibiotics successfully. Which virulence factors make these bacteria particularly virulent in regards to causing UTIs? A. Adhesins and hemolysin HlyA B. K1 capsules C. STa and LT1 toxins D. Stx1 and Stx2 toxins E. LPS and urease

A. Adhesins and hemolysin HlyA

The administration of Tocilizumab was effective in treatment plans that included which of the following? A. Antivirals B. Suppurative Treatment C. Stimulants D. Anti-inflammatory Supplements

A. Antivirals

A two-year-old of Mediterranean descent presented with enlarged, distorted cheekbones and signs of growth retardation. A CBC revealed low hemoglobin, low MCV, and excess iron. Additionally, her family's medical history showed a lineage of an autosomal homozygous genetic disorder. What kind of illness does this child have? A. Beta-thalassemia major B. Sickle cell anemia C. Alpha-thalassemia minor D. Immunohemolytic anemia E. Rickets

A. Beta-thalassemia major

Which of the following is commonly increased in patients with COVID-19? A. Cytokines B. Red Blood Cells C. Platelets D. CD4

A. Cytokines

A 30-year-old male suffers a knee injury that requires him to need surgery. In preparation for surgery, he undergoes testing, which includes; blood work, an EKG, chest x-ray, and a complete physical. After having his blood work performed, his PCP informs him that his blood work reveals anemia that is concerning. His PCP orders additional tests that reveal the man is heterozygous of β+ for β-thalassemia. The patient indicates that he has not noticed any changes in symptoms. He only reports mild fatigue from time-to-time, but attributes it to his busy work and his family schedule. What could be the cause of the patient's lab findings given his lack of symptoms? A. Elevated levels of HbF B. The hemoglobin ∝ and β- globin chains are in a 1:1 ratio C. Increased HbA/HbF ratio D. Increased concentration of HbC molecules in patient's RBC E. Deletion HPFH mutation, removing the entire 𝛿- and β-gene, leading to a decrease in HbF

A. Elevated levels of HbF

A pig farmer enters the clinic with complaints of abdominal pain and tenderness, watery stools with blood and pus for the past few days. He claims it most likely isn't food related since his wife and kids have been eating the same thing and had no issues that week. Upon further examination and one stool sample later, it has been revealed that his feces contain large trophozoites with cilia and a large macronucleus. What might the route of transmission of the suspected species found in his feces be? A. Fecal-oral B. Respiratory C. STD D. Direct skin penetration E. Arthropod bite

A. Fecal-oral

A 31-year old woman has just given birth to a child and is having trouble breastfeeding. When tests are performed, results show a lack of lactose and an excess of glycoproteins. Which enzyme catalyzes the last step of the defective pathway, and which subunit appears to be missing from the enzyme? A. Lactose synthase; α- lactalbumin B. Lactose synthase; galactosyltransferase C. Lactose dehydrogenase; α-lactalbumin D. Epimerase; galactosyltransferase E. Epimerase; α-lactalbumin

A. Lactose synthase; α- lactalbumin

A 28-year-old female patient returns to the Emergency Department after being previously diagnosed with Hemolytic Uremic Syndrome. She now complains of moderate/severe chest pain and shortness of breath accompanied by generalized weakness. Following some laboratory analysis, the physician determines that the patient is now suffering from a condition in which her RBC's are being destroyed as they pass through the small vessels of her body. Considering the patient's medical history, physician's new findings, and presentation of new symptoms, what extrinsic hemolytic hemolysis might the patient be suffering from? A. Microangiopathic hemolytic anemia (MAHA) B. Thrombotic Thrombocytopenic purpura C. HELLP syndrome D. Hereditary Spherocytosis (HS) E. Hereditary Elliptocytosis (HE)

A. Microangiopathic hemolytic anemia (MAHA)

A 23-year-old woman comes into the free health clinic with her boyfriend to get screened for any sexually transmitted infections. She has a history of several STDs in the past. The routine screening ordered for this woman included screening for Trichomonas vaginalis infection. Which one of the following would you recommend as the preferred screening test for detecting T. vaginalis infection? A. Microscopic examination for trophozoites B. Growth on media C. Gram staining D. Dark field microscopy E. Whiff/Amine test

A. Microscopic examination for trophozoites

One must be weary when trying to research information on SARS-CoV-2 because many of the published papers are? A. Not peer reviewed B. Inconsistent with data from the CDC C. Using longitudinal research to formulate the data D. Not receiving funding from the national institute of health (NIH)

A. Not peer reviewed

A 32 year old woman has recently given birth to a seemingly healthy baby boy. 48 hours after delivery, the baby started to show signs of yellowing of the skin and sclerae of the eyes. A total bilirubin lab test shows elevated levels of total bilirubin and an indirect bilirubin lab test shows elevated levels of unconjugated bilirubin. The baby was diagnosed with neonatal jaundice. What is the most likely treatment for neonatal jaundice in this patient? A. Phototherapy B. Supportive therapy C. Sulfa antibiotic therapy D. No effective therapy is known E. Steroid therapy

A. Phototherapy

A thirty-year-old female patient presents to the physician's office with influenza-like symptoms. Patient states that she returned from Costa Rica about 7-9 days ago. Patient also states that her symptoms developed further with characterized abdominal pain, vomiting of bile, severe diarrhea, and rapid dehydration. The physician orders RDT, which results in antigen detection seen in the figure below. Patient was diagnosed with malignant tertian malaria and was prescribed halofantrine and lumefantrine. The condition is most likely caused by which of the parasites listed below? A. Plasmodium falciparum B. Plasmodium ovale C. Plasmodium malariae D. Plasmodium knowlesi E. Plasmodium vivax

A. Plasmodium falciparum

What antibody responses were analyzed from the blood collected from COVID-19 patients? A. SARS-CoV-2-specific antibodies B. SARS-CoV- specific antibodies C. IgE D. IgA

A. SARS-CoV-2-specific antibodies

A 69 year old male hikes the Afghanistan mountains to visit the local village medical man. The patient waited a month before making the trip to the doctor to see if he would get better on his own. He had an ulcer on his right hand that was intensely pruritic, ulcerated, and crusted on the perimeter. The village medical man sees the characteristic ulcer and treats the patient with an injection of a pentavalent antimonial compound, sodium stibogluconate (Pentostam). Which of the following is the most likely vector that caused the patient's ulcer? A. Sandfly B. Tsetse fly C. Ixodes tick D. Blackflies E. Anopheles mosquito

A. Sandfly

A 25 year old male patient presents to a clinic in Nepal one rainy day with complaints of diarrhea and abdominal cramping having lasted a week. The patient shares that he has been unable to eat recently, but usually enjoys fresh salads he buys weekly from his local farmers market. Having seen these symptoms in several other patients this week, you suspect Cyclospora cayetanensis due to a local contaminated crop of spinach. A wet mount is necessary for diagnosis. Presence of which microscopic results would confirm the diagnosis of Cyclospora cayetanensis? A. Spherical oocysts 8-10 μm in diameter B. A large trophozoite with a pseudopod C. Flask shaped oocysts D. Elliptical oocysts 15-25 μm in diameter E. Oocysts containing five sporocysts

A. Spherical oocysts 8-10 μm in diameter

Which of the following is not an antigen-presenting cell? A. T-cell B. B-cell C. Macrophage D. Dendritic cell

A. T-cell

A researcher is studying the digestion of proteins in the stomach. The researcher decides to focus the study on the activity of pepsin, an enzyme in the stomach which initiates digestion, at different levels of pH. During one observation of the stomach acid, it is noted that pepsin is inactive. Which of the following is the explanation for the inactivation of pepsin? A. The stomach acid was too basic B. The stomach acid was too acidic C. The stomach acid was between 2.0 and 3.0 D. All of the above E. None of the above

A. The stomach acid was too basic

What neurological symptoms is SARS-CoV-2 LEAST likely to be associated with? A. Tingling in extremities B. Damaged blood brain barrier/loss of tight junctions C. Neuroinflammation of the brain parenchyma D. Stroke/acute neurotrauma E. Edema

A. Tingling in extremities

A 25 year old female comes to the clinic complaining of vaginal itching, discharge, and discomfort with urination. She tells the physician that she is sexually active and has been experiencing these symptoms for the past 7 days. The gynecological exam reveals a strawberry cervix and thin, frothy, and green vaginal discharge that is malodorous. The vaginal pH is 5.3. The physician prescribes her metronidazole. What parasite is most likely causing this patient's condition? A. Trichomonas vaginalis B. Toxoplasma gondii C. Trypanosoma brucei D. Entamoeba histolytica E. Giardia

A. Trichomonas vaginalis

RDV is potently antiviral against SARS-CoV-2 in primary human lung cultures with a selectivity index of more than 1,000. A. True B. False

A. True

According to Article #6, what cell type may not be the most appropriate to characterize the antiviral activity of Remdesivir? A. Vero E6 B. HAE C. Calu3 2B4 D. GS-44154

A. Vero E6

A 22-year-old female presents to her primary care physician of a fever of 100.1 degrees Fahrenheit, abdominal pain and vomiting. Upon physical examination, the patient was also found to have rose spots on her chest and abdomen. The patient mentions that two weeks prior to her appointment, the patient had returned from a vacation to India with her family. What is the initial pathology of the bacteria causing the patient's symptoms? A. invasion of the small intestinal M cells B. toxin release and activation of adenylate cyclase C. overgrowth of the bacteria in the colon D. attachment to the surface of CD4+ T-cells E. release of cytotoxins

A. invasion of the small intestinal M cells

Patient comes to the hospital with a fever, headache, and redness. When the physician looked at the patient white blood cell differentials, he noticed that the patient had an early, acute inflammatory reaction when he noticed elevations of which leukocyte and its granules contained lysosomal hydrolase and myeloperoxidase ? A. polymorphonuclear leukocyte B. Basophil C. monocyte D. eosinophil E. macrophage

A. polymorphonuclear leukocyte

What does low RBC count indicate?

Anemia bleeding bone marrow failure

A swine farmer in Kansas was working with his live stock on a particularly hot day. Having run out of water, he quickly scooped some water from the animals water supply before making the lengthy trip to refill his bottle at his house. Some time later he began to experience some abdominal pain, watery stool with blood and pus, and anorexia. What is he infected with? A) Salmonella Typhi B) B. Coli C) C. Coli D) Shigella sonnei E) E. Coli

B) B. Coli

A 23-year-old male presents to his PCP with complaints of chills, recurrent febrile attacks, and swollen genitalia. Swelling is also noted upon palpation of posterior cervical nodes and examination of his upper extremities reveal lymphangitis. Patient reports that he had been backpacking through Southeast Asia approximately 5mo ago. Peripheral blood smear reveals presence of sheathed microfilariae. What is the organism responsible for the patient's symptoms? A) O. volvulus B) B. malayi C) L. loa D) T. trichuria E) M. ozzardi

B) B. malayi

5. A 74 year old male was admitted to the ICU after being diagnosed with SARS-CoV-2. His labs showed that he had elevated inflammatory markers and imaging tests revealed that he had pleural line thickening. The hospitalist assigned to him presented his clinical case to her residents. She asked them to review literature and research what drugs could be repurposed to inhibit the entry of SARS-CoV-2. What receptor would need to be targeted to inhibit the entry of SARS-CoV-2? A. ICAM-1 B. ACE-2 C. CD-4 D. α1-adrenergic E. TLR-4

B. ACE-2

A 23 year old female presents to her primary care provider complaining of fatigue and decreased exercise tolerance. She states that she recently became vegetarian and reports a heavy menstrual cycle with an exacerbation of her symptoms. The PCP suspects Iron deficiency anemia and orders an iron study, which of the following lab results would confirm this diagnosis?

B.

Which percentage most likely represents the current CFR in Egypt? A. 20% B. 6% C. 2% D. 16%

B. 6%

What is the current recommendation regarding the length of appropriate isolation for those who test positive of SARS-CoV-2 RNA? A. 2-7 days B. 7-14 days C. 14-28 days D. 28-36 days

B. 7-14 days

A 44-year-old female presents to her gynecologist with malaise, fatigue, shortness of breath, and severe pain in the pelvic area. Although her menstrual cycles are regular, and she suffers from dysmenorrhea and menorrhagia. Lab tests primarily reveal a hemoglobin level of 10 g/dL (normal: 12-16 g/dL female), a hematocrit level of 34% (normal: 37-47% female), and an MVC level of 73 fL (normal: 80-95 fL). An ultrasound reveals a benign mass in her uterus that is exerting pressure on her urethra and bladder, leading to difficulty and pain in emptying urine, thus causing frequent UTIs. Before resorting to surgical options to remove the fibroid, the gynecologist wants to try other treatment methods to help the patient's symptoms. Which of the following treatments is best for this patient? A. Blood transfusion B. A diet rich in meat and dark, leafy greens C. Treatment with hydroxyurea D. A diet which avoids consumption of fava beans E. Treatment with antibiotics

B. A diet rich in meat and dark, leafy greens

What significant result did the Anti-malarial Drugs (Chloroquine and Hydroxy-Chloroquine) Meta-Analysis find when HCQ & AZM were used in combination to treat COVID? A. Adverse events, severe disease, duration of hospitalization favored the combination of drugs B. A significant increase in mortality rate resulted C. Resolution of cough and fever significantly improved D. The combination of drugs significantly reduced the severity of the cases

B. A significant increase in mortality rate resulted

A young female comes into the clinic concerned about a large ulcerative, red papule that is extremely pruritic. After asking about her travel history, it was discovered she went on a trip to Africa a month ago. The doctor diagnoses her with a blood and tissue infecting protozoa causing cutaneous leishmaniasis. In order to confirm this, what stage of the protozoa's life cycle must be seen in the lab diagnosis? A. Promastigote B. Amastigote C. Trypomastigote D. Epimastigote E. Trophozoite

B. Amastigote

Based on the information presented in the article, what clinical manifestation may Tocilizumab have a positive effect on in patients with a confirmed COVID-19 diagnosis? A. Blood Pressure B. Arterial Blood Saturation C. Respiration Rate D. Functional Reserve Capacity E. Total Lung Capacity

B. Arterial Blood Saturation

What is the main group of diseases that Tocilizumab is used to normally treat? A. Cancer B. Arthritis C. AIDS D. Fungal Infections

B. Arthritis

Postmortem findings for a recently deceased HIV-positive 23 year-old patient showed evidence of multiple brain abscesses, trophozoites and cysts present in their brain. Three days before they passed away, they presented to the emergency department and the resident on duty collected several specimens from the patient and sent them to be cultured. These cultures had proven difficult to grow on agar plates specific for two different types of amebic organisms. Finally, the microbiologist was successful in recovering the specimen in tissue culture using a mammalian cell line. What was the causative agent and form of encephalitis that the patient had? A. Acanthamoeba; chronic granulomatous encephalitis B. Balamuthia; chronic granulomatous encephalitis C. Naegleria; acute primary amebic meningoencephalitis D. Naegleria; chronic granulomatous encephalitis E. Acanthamoeba; acute primary amebic meningoencephalitis

B. Balamuthia; chronic granulomatous encephalitis

What is the protocol that the study, "The Duration of infectiousness of individuals infected with SARS-CoV2" used to gather data and to screen out studies that was not within their inclusion criteria? A. PROSPERO B. CRR C. FINER D. PICOT E. PRISMA-RR

B. CRR

A 52-year-old female with a history of Stage I colorectal cancer presented to the ED with complaints of neuropathy in bilateral upper extremities and pallor that has been ongoing for the past week and gradually becoming more prominent. She does have a surgical history of colon resection to remove the tumor and gastrectomy for other GIrelated issues. Since her cancer diagnosis, she has only been eating vegetables, CBC was obtained that demonstrated anemia with HGB of 7.5. MCV was noted to be elevated at 112 fL/cell. Based on her clinical symptoms and lab findings, what is she likely deficient in and what is the main cause of her deficiency respectively? A. Folate Deficiency; Not eating meat B. Cobalamin Deficiency; Loss of parietal cell function and IF secretion C. Folate Deficiency; Loss of function of the mitochondrial enzyme methylmalonyl-CoA mutase D. Cobalamin Deficiency; Resection of the Colon E. Folate Deficiency; Loss of parietal cell function and IF secretion

B. Cobalamin Deficiency; Loss of parietal cell function and IF secretion

44. A person who just experienced a traumatic car accident presents to the hospital. The physician finds that the patient sustained a spinal cord injury and the upper cord involved in the voluntary portion of defecation reflex is blocked. The physician suggested administration of small enema after a meal to excite the actions of the upper cord since the basic cord for reflex is still intact in the patient. Which area of the spinal cord (crucial for defecation reflex) is still intact in this patient. A. Ventral horn B. Conus medullaris C. Dorsal horn D. White matter E. Grey matter

B. Conus medullaris

A 25-year-old graduate student has finished school for the semester and decides to take a vacation. He goes to Nepal but has a hard time getting to his hotel due to the intense rain from the airport to the hotel, which happens a lot this time of year. The next day he goes on a hike, improperly prepared. He gets thirsty and pulls water from a local stream but treats the water with chlorine tablets before he drinks it. Later, he develops some mild nausea and watery diarrhea. What is the most likely organism the student has? A. Crytosporium parvum B. Cyclospora cayetanensis C. Giardia duodenalsis D. Dientamoeba fragilis E. Toxoplasma gondii

B. Cyclospora cayetanensis

Which of the following laboratory methods were used to measure SARS-CoV-2 specific antibodies? A. Complement fixation test B. ELISA C. Agglutination test D. Fluorescent antibody test E. Neutralization test

B. ELISA

A 22-year-old female patient visits the emergency department due to having a "pain crises." Her history shows that she suffers from sickle cell disease and is presenting today with severe body pains. Upon examination of her peripheral blood smear, irreversibly sickled cells can be visualized. Her blood work is indicative of hemolytic anemia, and reticulocytosis. This patient has increased susceptibility of which of the following? A. Falciparum malaria B. Haemophilus influenzae C. Acute Osteomyelitis D. Hypobilirubinemia E. Decreased occlusion of vascular beds

B. Haemophilus influenzae

A 22-year-old male is treated with sulfonamides for a recent Urinary Tract Infection. A few days later he experiences fatigue, jaundice and dark urine and goes in for a follow up visit the next day. The doctor notices that he has a pale appearance and conducts a few lab tests. The results showed that his hematocrit is at 24% and his spleen is enlarged. The doctor then performs a crystal violet staining of the peripheral blood smear. What was found in this smear that indicates the diagnosis? A. Reticulocytes B. Heinz Bodies C. Chronic Hemolysis D. Phenacetin E. Young RBCs

B. Heinz Bodies

A few months ago, a farmer in sub-Saharan Africa switched over to an organic fertilizer that is feces-based. After a long morning, the farmer's 10-year-old daughter danced around barefoot in the fields and then went straight to soccer practice. Once she was home, she immediately took off her cleats because her feet were so itchy. A few weeks later, she complained of abdominal pain, diarrhea, loss of appetite, fatigue, and shortness of breath. What was she infected with? A. Entamoeba histolytica B. Hookworm C. Pinworm D. Trypanosoma brucei gambiense E. Onchocerca volvulus

B. Hookworm

Which choice outlines the order of data selection in systematic review? A. Inclusion, identification, screening, eligibility B. Identification, screening, eligibility, inclusion C. Identification, eligibility, screening, inclusion D. Eligibility, identification, screening, inclusion

B. Identification, screening, eligibility, inclusion

Which of the following immunological responses is likely to be seen in patients infected with COVID-19? A. Increase number of total T-cells, CD4+ T cells, CD8+ T cells, and NK cells B. Induced differentiation of Th17 cells caused by an increase in the level of IL-6 C. Elevated lymphocyte count D. Deactivation of the innate immune system E. All the above

B. Induced differentiation of Th17 cells caused by an increase in the level of IL-6

What reason does the author of this article mention is a reason there is a very high rate of infections globally? A. Altered spike protein binding affinity B. Lack of prior immunity C. People not wearing masks D. Asymptomatic carrier spread E. Lack of immediate government response

B. Lack of prior immunity

A 28-year-old man comes in for an appointment with his primary care physician with complaints of fever, urticaria, and severe pruritus. He had moved to the United States 6 months ago from the Volta river basin where he worked near the streams. After physical examination, subcutaneous nodules were found on the head and neck, and blood tests revealed eosinophilia. Which laboratory test would confirm the diagnosis for the suspected organism? A. Flooding the nodule with water to recover larval worms B. Microfilariae in skin snip preparations C. Cysts in a stool sample D. Protozoa present in a wet prep E. Trypomastigote stage in a blood smear

B. Microfilariae in skin snip preparations

Bryce cut his vacation to South America short because he started feeling muscle pains, photophobia, nausea and vomiting. When he returned home, he sprung a fever of 105° F, began sweating and shaking, but the symptoms passed. After 48 hours he began feeling the same symptoms and decided to go to the doctor. The doctor suspects "benign tertian malaria." If this were true, what would be the optimal time to draw blood for a blood film? A. At the onset of symptoms B. Midway between paroxysms C. At the height of a paroxysm D. After photophobia symptoms begin E. A blood film can only be taken after death

B. Midway between paroxysms

A patient is seen in the clinic showing signs of fatigue, paleness, jaundice, and dark urine. Upon collection of a blood smear, Heinz bodies were noted. If the patient is suspected to have glucose-6-phosphate dehydrogenase deficiency, symptoms are occurring directly due to reduced production of which of the following? A. NADH B. NADPH C. ATP D. FADH2 E. O2

B. NADPH

A 22-year-old male patient presents to the Emergency Department with complaints of fatigue, weakness, chest pain, and headache. The patient explains that he has been getting much more exercise recently due to joining a church group basketball league. The patient also explains that he has been diagnosed with anemia. Which laboratory diagnostic test would the physician most likely order first, and what critical distinction might the results of the lab indicate to the physician? A. Peripheral Blood Smear; Increased RBC destruction or bleeding B. Reticulocyte Count; Primary failure of RBC production C. Erythrocyte Index; percentage of total number of RBC's D. Reticulocyte Count; Cause of anemia E. Peripheral Blood Smear; presence of hypoproliferative anemia

B. Reticulocyte Count; Primary failure of RBC production

A 10 year old male presents to the ER with complaints of abdominal pain, cramping, and watery but non-bloody diarrhea. His parents mention that he was recently vacationing with them and had swam in chlorinated, but unfiltered water. What is the best method of definitive diagnosis for this parasite? A. Histological examination of bowel tissue B. Stool analysis for O & P C. Blood analysis D. Electrolyte panel E. Diagnostic treatment with oral antimicrobials

B. Stool analysis for O & P

What do mast cells immediately release from their granules when they are activated? A. IL-1B, IL-6 and CCL2 B. TNF-a, histamine and proteases C. TNF-a, tryptase and GM-CSF D. Histamine, tryptase and proteases

B. TNF-a, histamine and proteases

What process of SARS-CoV-2 does the drug, Remdesivir (RDV) and its analog GS-441524 target to inhibit the virus' function? A. attachment B. replication C. penetration D. uncoating

B. replication

A 24-year-old female reports to her OB-GYN with a chief complaint of frothy yellowish, green discharge. Reveals having multiple sexual partners over the past couple years. Upon examination a strawberry cervix was noted with the causative agent resembling a flagellated protozoan on a wet prep. Which of the following is the most likely genital infection contracted by this patient? A. Vulvovaginal candidiasis B. Trichomonas vaginalis C. Gardnerella vaginalis D. Treponema pallidum E. Herpes Simplex virus 2

B. Trichomonas vaginalis

A 27 year old male presents to the Emergency Department with abdominal pain and diarrhea. After a series of tests, the physician discovers flask-shaped ulcerations of the intestinal mucosa. A microscopic examination of the patient's stool confirms the physician's suspicions. What stage of the microorganism is responsible for the formation of these ulcers? A. Oocysts B. Trophozoites C. Sporozoites D. Cysts E. Sporocysts

B. Trophozoites

In article #6, Which cell line had the highest viral yield after being infected with SARS-CoV-2? A. HAE B. Vero E6 C. Calu32B4 D. Huh7

B. Vero E6

Which of the following is NOT a potential side effect of Tocilizumab? A. Headaches B. Vomiting C. Rash D. Respiratory infections

B. Vomiting

A 20-year-old female college student is exercising at the campus gym when she experiences difficulty breathing and palpitations. An employee, noticing her pale complexion and seemingly extraordinary exhaustion, escorts the student to the school health center. Blood work reveals reduced RBC mass. An inborn error in what type of metabolism could cause the student's blood disorder? A. amino acid B. carbohydrate C. porphyrin D. steroid E. purine/pyrimidine

B. carbohydrate

What does elevated neutrophils (PMNs) indicate?

Bacterial infection

A 54-year-old male reports to his new PCP with complaints of fatigue, weakness, and chest pain. In the appointment after reviewing the man's chart, the doctor finds that he had a transcatheter aortic valve replacement about a year ago. Suspecting the man to have anemia the physician orders for a blood smear to confirm. What cells in addition to RBCs would be found in the blood smear? A. Bite Cells B. Sickled Cells C. Burr/Helmet Cells D. Spherocytes E. Heinz Bodies

C. Burr/Helmet Cells

A 2-year-old female is taken to the pediatrician by her mother as she has been experiencing diarrhea for the last three days and has been increasingly fatigued. Her mother is concerned that she may be dehydrated. Her mother informs the physician that she has been going to daycare daily for the last 6 months and there are a few other children with similar symptoms. What is the incubation period of the bacteria causing her symptoms? A. 1-5 days B. 3-4 days C. 6-72 hours D. 2-7 hours E. 3-7 days

C. 6-72 hours

A patient who came to the hospital complaining of abdominal pains, cramps, steatorrhea, foul-smelling watery diarrhea. He said that symptoms started around 2 week after eating uncooked broccoli at a restaurant near a lake in Canada. The physician suspected that the patient might have gotten in contact with water that has been contaminated with feces. He ran the patient through different lab tests including stool O & P. After looking at lab results, the patient was given metronidazole. Which organism do you think the patient was infected with? A) E.Histolytica B) Trichomonas C) Giardia D) ETEC E) EHEC

C) Giardia

A 32-year-old patient presents to her primary care physician with a fever, chills, and swollen lymph nodes. The patient denies any recent travel outside of the USA, but further prompting reveals she traveled to Central Africa one year prior. Which of the following is an accurate part of the causative agent's life cycle? A) The larvae will hatch in the small intestine and later migrate to the large intestine B) The larvae will grow in the alveoli C) The larvae will mature in the lymphatic system D) The larvae will migrate to cecum and penetrate mucosa E) The larvae will mature in the retroperitoneal space

C) The larvae will mature in the lymphatic system

The rapid review of the duration of infectiousness of SARS-CoV-2 published by Walsh, Kieran et al. included how many relevant studies? A. 5 B. 7 C. 15 D. 21 E. 24

C. 15

How many peer-reviewed studies were selected for the meta-analysis? A. 2 B. 8 C. 17 D. 28

C. 17

A 29 year old female goes to the doctor to find out what her blood type is. She wants to see if she could donate blood to a sibling needing a transfusion. They tell her she has a B antigen and has anti-A serum antibodies. What blood type does the sibling need to be in order to be able to receive her blood? A. Rh negative B. A C. AB D. O E. Rh positive

C. AB

A 58-year old woman presents to her PCP with concerns of food "feeling stuck" in her throat during and following a meal. Her physician is concerned that she may have somehow damaged part of her myenteric plexus. What term describes her symptoms, and if left untreated what may it lead to? A. Constipation; Megaesophagus B. Achalasia; Megacolon C. Achalasia; Megaesophagus D. Achalasia; Pernicious Anemia E. Gastritis; Peptic Ulcer

C. Achalasia; Megaesophagus

Mast cell activation through SARS-CoV-2 occurs through which mechanism? A. Binding to immunoglobulin E-receptors B. Binding to stem cell factors C. Binding of toll-like receptors D. Immunoglobulin G-receptors E. MAP kinase signal transduction

C. Binding of toll-like receptors

A 29 year old male presents to his primary care physician for a scheduled complete blood count (CBC) following a previous annual visit. He has a history of having unprotected sex with multiple partners over the past 6 months and has not traveled outside of the country in the last year. His CBC results come back and show there is a decrease in his Leukocyte count with normal Erythrocyte, Hematocrit and Hemoglobin levels. Which of the following is NOT a probable cause of the decrease in his leukocytes? A. Chemotherapy/Radiation B. Bone Marrow Diseases C. Bone Fractures D. Autoimmune Disorders E. HIV/AIDS

C. Bone Fractures

In Article #6, what enzyme did the researchers alter in mice to show RDV is active against SARS-CoV-2 in vivo? A. RdRp+/+ B. PenH-/- C. Ces1c-/- D. mACE2+/+

C. Ces1c-/-

A 32-year-old male patient presents to the clinic with complaints of abdominal cramps, nausea, vomiting, and watery diarrhea. The patient has recently traveled to Costa Rica for a business trip and started experiencing these symptoms two days after the trip. What organism is most likely responsible for these symptoms? A. Clostridium Difficile B. Yersinia Enterocolitica C. Enterotoxigenic Escherichia coli D. Shigella E. Entamoeba Histolytica

C. Enterotoxigenic Escherichia coli

A 32 year old female presents to her physician complaining of chronic itching, abdominal pain, nausea and loss of appetite. She also states that she has noticed red marks developing under the surface of the skin around the area where the itching has been occurring. She says that she spends a lot of time outside camping and hiking with her dog. Further examination reveals that she is suffering from an infection caused by the parasite Necator americanus. Which type of blood cell will most likely be active at the site of infection? A. Neutrophil B. Basophil C. Eosinophil D. Erythrocyte E. Thrombocyte

C. Eosinophil

A 54-year-old male, who recently visited his family in Germany, presents to his PCP with abdominal pain, bloating, diarrhea, nausea, and vomiting. He tells the physician that he lost weight recently due to loss of appetite and complains of light, foul-smelling stools. He also recently underwent a Whipple procedure to remove the head of his pancreas due to malignancy. Which of the following should the physician expect to see increased in the patient's stools? A. Blood B. Entamoeba histolytica cysts C. Fat D. ETEC E. Leukocytes

C. Fat

A 1 y.o. male child presents to his PCP with general weakness, fatigue, loss of appetite, and irritability. Parents have been supplementing diet with PediaSure mix. Normal diet is well balanced. Upon testing, his red blood cells appear microcytic and hypochromic. Further questioning reveals that he has been teething and currently uses a newly thrifted 80 year old antique crib. What deficiency is causing the patient's symptoms? A. Vitamin B6 B. Folate C. Heme D. B12 E. G6PD

C. Heme

A 3-year-old female presents to the ER with initially watery diarrhea that has become bloody, abdominal pain, and signs of severe dehydration. The patient's parents state that symptoms started about 3 days after they gave the child fruit juice from a local farm. If the family refuses treatment, what condition still remains a risk for the patient? A. Neonatal Meningitis B. UTI C. Hemolytic Uremic Syndrome D. Traveler's Diarrhea E. Dysentery

C. Hemolytic Uremic Syndrome

Researchers are studying the effects of anti-malarial drugs in treatment of COVID-19 patients in a randomized clinical trial. What is the most likely reason the researchers chose to use Hydroxy-Chloroquine instead of Chloroquine? A. Hydroxy-Chloroquine is cheaper and more accessible than Chloroquine B. Hydroxy-Chloroquine inhibits Angiotensin Converting Enzyme 2 whereas Chloroquine inhibits the release of viral particles C. Hydroxy-Chloroquine is more soluble and less toxic than Chloroquine D. Hydroxy-Chloroquine is more well-known than Chloroquine

C. Hydroxy-Chloroquine is more soluble and less toxic than Chloroquine

The drug Tocilizumab is used to treat patients with COVID-19. Which of the following inflammatory cytokine receptors does it competitively bind to? A. TNF-alpha B. G-CSF C. IL-6 D. IP10 E. MIP1A

C. IL-6

What is the receptor that Tocilizumab binds to? A. mAch B. HER2 C. IL-6 Receptor D. ERBB3

C. IL-6 Receptor

A 28 year old male reports to his primary care physician after coming down with a fever. He said that the fever comes and goes every few days. Because he has just traveled to a subtropical environment about two months ago, his PCP recommends obtaining blood for lab testing, including a blood smear. The lab test results show infection with a rosette schizont, as well as bar and band forms of the parasite appearing in the host cell. Which type of cell is the host cell for the parasite? A. Young, pliable erythrocytes B. Myocytes C. Mature erythrocytes only D. Sickle cells E. Spherocytes

C. Mature erythrocytes only

A 12 year-old female presents to the emergency department with fatigue and pallor. Upon H/P exam she reports that she began menstruating 5 months ago and doesn't eat any meat, and iron deficiency is suspected. A routine reticulocyte count is performed and is found to be particularly low. Which test would further confirm iron-deficient anemia based on the size of erythrocytes? A. Peripheral blood smear B. Peripheral blood iron indices C. Mean corpuscular volume (MCV) D. Bone marrow aspirate E. Nutritional exam

C. Mean corpuscular volume (MCV)

What domain did the key residue T680 lie in that allowed the coronavirus family to be distinguished from other viruses? A. Nsp7 B. Nsp8 C. Nsp12 D. Nsp11

C. Nsp12

A 25-year-old, African American male presents to the Emergency Department with severe pain in his bones. It was later discovered that he was suffering from a vaso-occlusive pain crisis. His blood smear indicates needlelike fibers within red cells, and a lab evaluation shows increased MCHC levels which facilitates the formation of these needlelike fibers. Which of the following would be the most likely cause of this disease? A. G6PD Deficiency B. Frameshift mutation affecting spectrin C. Point mutation in β-globin D. Chain terminator mutations E. Absent synthesis of α-globin chain

C. Point mutation in β-globin

A 25-year-old male presents to the emergency room with complaints of abdominal pain, cramping, and colitis with diarrhea. The patient indicates that he has recently returned from a mission trip in Africa. He states that he had been diligent about drinking water from only sanitized and packaged water bottles. However, he does recall getting an iced drink from a local vendor toward the end of his trip. Lab testing and stool samples indicate that the patient has been infected with an organism that leads to flask-shaped ulcerations of the intestinal mucosa and extensive local necrosis in the large intestine. The basis for the extensive local necrosis and tissue destruction is attributed to which of the following? A. Invasion of trophozoites into the mucosal lining B. Trophozoite attachment to intestinal villi C. Production of a cytotoxin D. Sexual and asexual reproduction of organism in the intestinal epithelium E. Trophozoite multiplication by binary fission

C. Production of a cytotoxin

A recent immigrant and dairy farmer from northern Europe, visits the outpatient clinic for the first time. She's experiencing both nausea and vomiting, but decided to come in due to pharyngitis and arthralgia. Further questioning reveals she has experienced no fever or diarrhea. Physical examination also reveals erythema nodosum on the front of her legs below the knee. The physician orders a stool sample, and the pathogen responsible grows on Yersinia-selective agar. What complication is possible following infection? A. Guillain-Barre B. Hemolytic-uremic syndrome C. Reactive arthritis D. Toxic megacolon E. Bloody diarrhea

C. Reactive arthritis

A patient sick with COVID is talking to their doctor about the first day they experienced symptoms. The patient is struggling to remember whether their symptoms started 3 or 4 days previously. What is this phenomenon called? A. Allocation Bias B. Ascertainment Bias C. Recall Bias D. Misclassification Bias

C. Recall Bias

The active site of SARS-CoV-2 is most similar to what other coronavirus? A. HCoV-OC43 B. MERS-CoV C. SARS-CoV D. HCoV-229E

C. SARS-CoV

A 30-year-old female visits her physician with complaints of fever, myalgia, and arthralgia. She tells him that she used to have an ulcer on her arm a few months ago. In her travel history, she mentions that she has visited Western Africa and Central America in the last six months. The doctor notes swelling of the posterior cervical lymph nodes in the patient. What is the most likely pathogen responsible for the symptoms experienced by the patient? A. P. falciparum B. T. Cruzi C. T. Brucei Gambiense D. T. Brucei Rhodesiense E. Dracuncula medinensis

C. T. Brucei Gambiense

3. If the R0 of a particular virus is 3, what does that tell us about the infectivity and, therefore, the potential spread of this virus? A. This virus is not very infectious and will eventually die out. B. This virus is moderately infectious but will remain stable in the population and will not lead to epidemic spread. C. This virus is highly infectious and will eventually lead to an outbreak.

C. This virus is highly infectious and will eventually lead to an outbreak.

Which of the following is the overarching goal of the paper? A. To determine the prevalence of asymptomatic cases of SARS-CoV-2 B. To investigate differences in infectivity across age groups C. To gain further understanding of the adaptive immune responses to SARS-CoV-2 D. To further understand the pathophysiology of SARS-CoV-2 E. To investigate the mechanism of SARS-CoV-2 binding to ACE2

C. To gain further understanding of the adaptive immune responses to SARS-CoV-2

Which of the following enzymes is correctly matched to the heme synthesis disorder most likely occurring in a patient who presents to the clinic with symptoms consisting of anxiety, light sensitivity, and increased facial hair? A. Protoporphyrinogen oxidase, acute intermittent porphyria B. Delta-ALA dehydratase, Lead poisoning C. Uroporphyrinogen decarboxylase, porphyria cutanea tarda D. Uroporphyrinogen decarboxylase, congenital erythropoietic porphyria E. Cytochrome b5 reductase, methemoglobinemia

C. Uroporphyrinogen decarboxylase, porphyria cutanea tarda

Which patient population was used as the sample in this research paper? A. Patients who were infected with COVID-19 but exhibited mild symptoms B. Patients who were infected with COVID-19 but were asymptomatic C. Patients who were hospitalized with COVID-19 and on ventilator support D. Patients who recently recovered from COVID-19 E. Patients who had a co-infection with COVID-19 and influenza

D. Patients who recently recovered from COVID-19

How many studies were screened for the rapid review of the duration of infectiousness of SARS-CoV-2 published by Walsh, Kieran et al.? A. 197 B. 905 C. 842 D. 4,226 E. 729

D. 4,226

A 21-year-old male from Northern Europe presents to the ER complaining of fatigue, lightheadedness, and appears jaundiced. Upon physical examination there appears to be tenderness in the right upper quadrant of his abdomen. He claims the lightheadedness comes and goes, but it has become more debilitating over the last few days. An ultrasound is ordered along with a CBC. Ultrasound shows the presence of gallstones and CBC indicates slightly lowered hemoglobin and hematocrit. These lab findings point to a diagnosis of Hereditary Spherocytosis. What is responsible for the morphology of red blood cells in patients with Hereditary Spherocytosis? A. Inability to convert oxidized glutathione to reduced glutathione, resulting in reduced protection against oxidant injury B. Missense mutation in the B-globin gene, replacing a glutamine residue with a valine residue C. Splicing mutation that destroys the normal RNA splice junction and prevents the production of normal βglobin mRNA D. A frameshift mutation that results in the failure to produce proteins, resulting in destabilization and the shedding of membrane fragments E. Somatic mutation inactivating PIGA, resulting in a cellular deficiency

D. A frameshift mutation that results in the failure to produce proteins, resulting in destabilization and the shedding of membrane fragments

Mark, A 44-year-old avid runner complained to his wife on their 20th wedding anniversary about an unusual bone pain and weight loss. Given his interest in exercise coupled with a keen interest in ketogenic diet. They both instinctually attributed these symptoms to possibly be induced by his diet regimen and exercising. 120. However, three days later the pain progressively continued to worsen with developing fever, nosebleeds, and swollen lymph nodes. Upon visiting the ER, the attending physician decided to run a CBC blood test and conduct a bone marrow biopsy in order to make a differential diagnosis. The test results revealed that Mark has a 90% bone cellularity index (Hypercellular bone marrow) coupled with an excessive buildup of immature white blood cells. Which of the following is a plausible diagnosis the physician might be most concerned with? A. Aplastic anemia B. Subacute sclerosing panencephalitis C. Osteomalacia D. Acute myelogenous leukemia E. Hodgkin's lymphoma

D. Acute myelogenous leukemia

A 4 y/o boy arrives to the emergency department with his mother with complaints of a high fever and fatigue. During a physical exam, the physician notices a rash and edema around the boy's eyes and face. The patient's mother explains to the physician that they recently moved to the United States from Venezuela 3 days ago. What form in the life cycle of this parasite differs from the other Trypanosoma parasites and could help in confirming a diagnosis? A. Promastigote B. Epimastigote C. Trypomastigote D. Amastigote E. Trophozoites

D. Amastigote

The Anderson Family takes an annual trip to Nantucket, which they have been doing for the past ten years. About two- weeks after returning home, Mrs. Anderson begins to experience a moderate degree of malaise, a headache, chills, sweating, and fatigue that is not improving. She visits her doctor and later a blood smear is performed. The results reveal that her RBCs are infected with multiple ring forms. What would be the most appropriate form of treatment for this infection? A. Penicillin B. Chloroquine C. Clindamycin and Quinine D. Atovaquone and Azithromycin E. Pantoprazole

D. Atovaquone and Azithromycin

Tocilizumab is a drug that is usually prescribed to treat which types of disorders? A. Gastrointestinal B. Endocrine C. Respiratory D. Autoimmune

D. Autoimmune

According to Efficacy and Safety of Anti-malarial Drugs (Chloroquine and Hydroxy-Chloroquine) in Treatment of COVID-19 infection, what medication, when used in conjunction with Hydroxy-Chloroquine, increased mortality? A. Remdesivir B. Steroids C. NSAIDs D. Azithromycin

D. Azithromycin

A 19-year-old African American male presents to his family physician with complaints of fatigue and weakness. He appears uncharacteristically pale. This is the second trip to his family physician this week. In his visit 3 days ago, his physician diagnosed him with viral hepatitis. While suffering from liver pain during that visit, he had not been feeling fatigued. His physician suspects a type of hemolytic anemia brought on by the environmental trigger of a viral infection. Which of the following when viewed on a blood smear would confirm the diagnosis of this specific hemolytic anemia? A. Reticulocytes B. Anisocytosis of Red Blood Cells C. Howell-Jolly bodies D. Bite Cells E. Sickled Red Blood Cells

D. Bite Cells

Stress associated with COVID-19 is correlated with an exacerbation of all of the following diseases/disease symptoms EXCEPT? A. Headaches B. Asthma C. Sleeping disorders D. Depression E. Shortness of breath

D. Depression

Ian is a 24-year-old professional cyclist who has qualified to compete in the 2024 Summer Olympics in Paris for Team USA. As part of the International Olympic Committee's Anti-doping campaign, Ian is chosen for a random blood test to check for the use of performance enhancing drugs. His blood test results are below: Which of the following regulatory substances is responsible for regulating the formation and release of the blood component that is abnormal in Ian's CBC report? Which organ secretes this regulatory substance? A. Erythropoietin; thymus B. Thrombopoietin; kidneys C. Lymphopoietin; liver D. Erythropoietin; kidneys E. Reticulopoietin; bone marrow

D. Erythropoietin; kidneys

A patient showed up to her PCP concerned about a painful ulcer on her foot. When asked about her travel history, the patient said that they had not left the country recently, but they had gone on a mission trip to South Sudan a year ago. However, she doubted that she had gotten sick there, as she had not had any symptoms after her trip and the ulcer was a recent development. Which of the following would be the most appropriate to diagnose the patient? A. Ova and parasite stool test B. Blood smear C. Giemsa stained blood films D. Flooding the area with water E. Skin biopsy from ulcer

D. Flooding the area with water

A 45-year-old Caucasian male presents to his primary care physician complaining of diarrhea, fatigue and joint pain for the past month. He also mentions that he has been steadily losing weight for the past 6 months. He denies any recent changes to his diet. A PAS stain and an acid-fast stain are performed, which confirm the physician's diagnosis. Which of the following is a characteristic of the disease described by the patient's symptoms? A. FOXP3 gene mutation B. Purulent pseudomembranes C. Attaching and effacing (A/E) lesions D. Foamy macrophages with gram-positive microbes E. NKG2D and MIC-A interaction

D. Foamy macrophages with gram-positive microbes

A 58-year-old male accountant arrives at the doctor for his routine physical. He offers complaints of fatigue and feeling lightheaded. He attributes this to working extra hours at a local gun range. What is an accurate explanation regarding his condition? A. Decreased δ-ALA, decreased protoporphyrin IX B. Increased Porphobilinogen, increased Copropophyrinogen III C. Decreased δ-ALA, increased protoporphyrin IX D. Increased δ-ALA, increased protoporphyrin IX E. Decreased Porphobilinogen, increased Copropophyryinogen III

D. Increased δ-ALA, increased protoporphyrin IX

A 16-month infant presents to your pediatrician office with his parents. His parents report that he has been vomiting for the past 4 days with associated cough and coryza. The infant recently started going to daycare around 3 months ago, after his mother started a new job. You suspect that the causative agent is a naked, dsRNA virus. What is the mechanism of infection for this particular virus? A. Macrophages accumulate within the small intestine lamina and mesenteric lymph nodes. B. Direct invasion of epithelial cells. C. Use of adhesins. D. Infection and destruction of mature enterocytes in the small intestine. E. Release cytotoxins that cause direct epithelial damage.

D. Infection and destruction of mature enterocytes in the small intestine.

A 25 year old male presents to the Emergency Department with generalized weakness and fatigue. After an abnormal CBC, the physician orders a blood smear which reveals spherical erythrocytes. The patient is diagnosed with hereditary spherocytosis, a genetic disorder that causes an abnormal shape in erythrocytes. What is the function of the defective protein complex in erythrocytes? A. Express specific blood group antigens on the surface of erythrocytes B. Transport oxygen and carbon dioxide throughout the circulatory system C. Maintain the colloid osmotic pressure to prevent erythrocyte swelling D. Interact with spectrin filaments to anchor the plasma membrane to the cytoplasm E. Bind hemoglobin and act as an anchoring site for cytoskeletal proteins

D. Interact with spectrin filaments to anchor the plasma membrane to the cytoplasm

Private Ryan, a 21-year-old male serving in the United States army pays a visit to his primary care physician during his return home for the Thanksgiving holiday. His chief complaint to his doctor is that he had a red papule on his arm during his deployment in Iraq. After about 2 weeks, the papule began to ulcerate and gradually leaked a thin, serous material. Based on this clinical description, what vector did Private Ryan come in contact with? A. Dirt fly B. Tsetse Fly C. Ixodes Scapularis D. Lutzomyia Sandfly E. Anopheles Mosquito

D. Lutzomyia Sandfly

In this study, what was shown to increase reproductive variability of SARS-CoV2 in provinces of China? A. Temperature alone B. Humidity alone C. Environmental factors alone D. More than environmental factors E. Decreased human mobility

D. More than environmental factors

A 3-yr old male presents to the ER with fever, chest pain, difficulty breathing, and bluish tinge on fingers. It is determined the patient is experiencing acute chest syndrome and needs an emergency blood transfusion. A blood culture is performed and high levels of Salmonella are found in their system. What would this cause in the patient? A. Aplastic Crisis B. Hydroxyurea C. Heinz bodies D. Osteomyelitis E. Vitamin B12 Deficiency

D. Osteomyelitis

Ahmad, 33-year-old male, presents to the doctor with hemolytic anemia and yellowing of the sclera of his eyes. Eventually the doctor determined that the likely cause of his symptoms was an X-linked genetic defect in the gene that codes for glucose 6-P dehydrogenase. If the diagnosis is accurate, what is the pathway in which Glucose-6-Phosphate dehydrogenase is the rate limiting enzyme and what change in activity would be associated with this genetic defect? A. Pentose phosphate pathway; Increased NAPDH levels and decreased superoxide synthesis B. TCA cycle; decreased NADPH and increased NADH C. Pentose phosphate pathway; Decreased NADPH levels and Increased superoxide synthesis D. Pentose phosphate pathway; Decreased NADPH levels and decreased Cytochrome P450 monooxygenase E. Not enough information given

D. Pentose phosphate pathway; Decreased NADPH levels and decreased Cytochrome P450

A patient is told by their physician that they are lacking an enzyme that is essential for digesting a major protein found in the connective tissue of meats. This digestion allows for other enzymes to penetrate ingested meats, and without it, meats may be poorly digested. What is the enzyme and which protein fibers does it digest? A. Trypsin; collagen fibers B. Pepsin; elastin fibers C. Chymotrypsin; collagen fibers D. Pepsin; collagen fibers E. Trypsin; elastin fibers

D. Pepsin; collagen fibers

A 65-year-old female presents to the emergency room with complaints of fatigue, nausea, heartburn and severe abdominal bloating. After obtaining the patient's medical history, it is discovered that the patient has been suffering from long-term alcohol abuse. A form of gastric atrophy is suspected, which resulting condition would you expect to find and what complications could occur as a result of that condition? A. Achalasia; The ulceration of the mucosa B. Achlorhydria; Increased production of pepsin in the gut C. Sprue; Osteomalacia D. Pernicious anemia; Decreased B12 absorption E.Megacolon; Constipation

D. Pernicious anemia; Decreased B12 absorption

A 29 year old patient presents to his doctor with fever, chills, nausea, and vomiting. He says that he returned from a trip to Peru about six months ago. Upon return, he experienced a similar sickness, and was treated with chloroquine for malaria. His symptoms had cleared up after treatment and had subsided until the past 3 days. What organism is most likely responsible for this patient's symptoms? A. Plasmodium falciparum B. Plasmodium knowlesi C. Leishmania donovani D. Plasmodium vivax E. Babesia microti

D. Plasmodium vivax

Which of the following was not considered inclusion criteria for the analysis on the duration of SARS-CoV-2 by Walsh, Kieran et al.? A. Contact tracing performed B. Virus culture performed C. Human populations D. Publications that included animal studies E. Studies that only looked at the SARS-CoV-2 virus

D. Publications that included animal studies

What type of cells does SARS-COV-2 infect at first and what does it affect? A. SARS-COV-2 effects the bronchi. Its function is to produce mucous. B. SARS-COV-2 effects pneumatocysts. Its function is to produce mucous. C. SARS-COV-2 effects the bronchi. Its function is to produce surfactants. D. SARS-COV-2 effects pneumatocysts. Its function is to produce surfactants.

D. SARS-COV-2 effects pneumatocysts. Its function is to produce surfactants.

A 26-year-old female patient presents to the emergency department with bloody diarrhea (dysentery). The patient is a mother of 2 young children in daycare who have similar symptoms. What is a likely organism and category of the diarrhea? A. V. cholerae- enterotoxin B. V. cholerae- cytotoxin C. Shigella dysenteriae- enterotoxin D. Shigella dysenteriae- cytotoxin E. Shiga toxin- producing strain of E. coli (STEC)- enterotoxin

D. Shigella dysenteriae- cytotoxin

Which of the following transporters absorbs sodium across the basolateral membrane in the intestinal epithelium? A. Sodium-Glucose Co-transporter B. Sodium-Hydrogen Exchanger C. Sodium-Chloride Symporter D. Sodium-Potassium ATP-ase pump E. Sodium-Amino Acid Co-transporter

D. Sodium-Potassium ATP-ase pump

A 46-year-old male has presented to his primary care physician with digestive issues and serious pain in his heart. The patient has taken it upon himself to become a master of WebMD and diagnosed himself with esophageal cancer. After multiple tests and treatment with omeprazole, it was confirmed severe heartburn. After carefully educating the patient about avoiding fats and chocolate; what part of the patient's digestive tract is the poorest absorptive area only absorbing a few highly lipid soluble substances? A. Valvulae conniventes B. Large intestine C. Pancreas D. Stomach E. Small intestine

D. Stomach

A four-year-old male child presents to the pediatrician's office with his mother. The patient's mother states that her child has developed difficulty swallowing, impaired vision, and involuntary muscle spasms. Additionally, the patient's mother states that her child had several seizures in the past month. The doctor orders blood work which reveals mutation in the enzyme, called Hexosaminidase A (Hex A). The enzyme mutation is related to which of the following diseases? A. Fabry disease B. Krabbe disease C. Farber disease D. Tay-Sachs disease E. Niemann-Pick disease

D. Tay-Sachs disease

Which of the following outlines the relationship between temperature and humidity with transmission of SARS-CoV2? A. Temperature - negative relationship, Humidity - no relationship B. Temperature - negative relationship, Humidity- negative relationship C. Temperature - no relationship, Humidity - negative relationship D. Temperature - no relationship, Humidity- no relationship

D. Temperature - no relationship, Humidity - no relationship

A 13-year-old patient presents to his primary care physician with his mother with concerns of his diarrhea, bloating, loss of appetite, and weight loss over the past couple weeks. The mother states that the only recent change is that they switched to a ketogenic diet. What could be the cause of the patient's symptoms? A. The patient's sodium co-transporter is not functioning properly. B. The patient's unable to absorb his monosaccharides via facilitated diffusion. C. The patient's secondary active transport system is not functioning properly. D. The patient does not have enough functioning micelles. E. The patient is unable to directly absorb short-chain fatty acids.

D. The patient does not have enough functioning micelles.

A 25-year-old woman has recently returned to the United States after spending a week in Ethiopia. Over the next few days, she experiences chills and fever that reappear periodically and decides to visit the doctor. After a few laboratory tests, it was revealed that the red blood cells were oval shaped with a ragged cell border. There was also a presence of Schuffner dots throughout the cytoplasm. What laboratory test was used to identify this information and lead to a diagnosis? A. Rapid Diagnostic tests (RDTs) B. PCR C. Serological Test D. Thick and Thin Blood Films E. Drug Resistance Tests

D. Thick and Thin Blood Films

A 43-year-old man returned from a recent visit with his parents. He goes to his primary care physician with complaints of lesions. He mentions hunting in the woods of South Carolina with his parents and receiving many bug bites. After some testing, the physician found that the bacteria causing the lesions has a prominent capsule and a mucoid appearance of the colonies. The man is diagnosed with Donovanosis. What other disease has similar resemblance to this diagnosis? A. Enteroaggregative escherichia coli B. Proteus mirabilis C. Rickettsia rickettsii D. Treponema pallidum E. Yersinia pestis

D. Treponema pallidum

Which of the following is NOT a possible pathway that SARS-CoV-2 can enter into the brain based on the information presented in the article? A. Olfactory nerve B. Nose C. Lymphatic drainage D. Trigeminal nerve E. Defective blood brain barrier

D. Trigeminal nerve

A 53-year-old male is seen in his cardiologist's office for follow-up regarding history of atrial fibrillation and flutter. He had received 2 infusions of procainamide 2 weeks prior to this office visit. Upon general appearance, the patient appears pale and fatigued. During the physical, the cardiologist noted bilateral conjunctival icterus. Bloodwork revealed decreased RBC count, elevated reticulocyte count and decreased hematocrit. Which of the following describes the mechanism of hemolysis for this patient? A. IgM antibodies are produced and directed against RBC antigen I B. Drug binds to erythrocytes and acts as a hapten; antibodies are produced and directed against the drug C. IgG antibodies are produced and directed against RBC antigen i. D. True anti erythrocyte antibodies are produced and directed against RBC antigens including Rh. E. An antibody-drug complex forms and binds to the erythrocyte membrane to active complement

D. True anti erythrocyte antibodies are produced and directed against RBC antigens including Rh.

Upon arrival to the ED, an 11-year-old male underwent anaphylaxis. His mother reported that she brought him to the ED due to an acute onset of hives on his left upper arm and difficulty breathing immediately after he was stung by a bee while climbing a tree with his friends. She stated that the patient had not been stung by a bee before until recently. The patient's vitals are as followed: BP 85/60 mmHg, HR 120 bpm, RR 32 breaths/min, and body temperature 98.6 °F. Which of the following leukocytes is responsible for this patient's symptoms? A. Neutrophils B. Lymphocytes C. Eosinophils D. Monocytes E. Basophils

E. Basophils

Evidence reviewed by Walsh, Kieran et al. suggests that COVID-19 patients with mild illness are unlikely to be infectious after ___ days following initial symptoms. A. 6 B. 7 C. 8 D. 9 E. 10

E. 10

Evidence reviewed by Walsh, Kieran et al. suggests that patients with severe symptoms and those who are immunocompromised may be infectious for up to ___ days or more, after initial symptoms. A. 10 B. 12 C. 14 D. 16 E. 20

E. 20

A 27 year old man on a mission trip falls ill and meets the local doctor. He expresses complaints of profuse watery diarrhea with a strong odor and dehydration. Within 24 hours, the man had passed. The doctor has no way of testing the stool samples, if they did, what would they find? A. A spore forming gram negative rod B. A toxin producing gram negative bacillus C. A spore forming gram positive bacillus D. A protozoan parasite/ parasitic cysts and trophozoites E. A toxin producing gram negative curved rod

E. A toxin producing gram negative curved rod

COVID-19 activates mast cells to release several molecules that can have a proinflammatory response. Which of the molecules is released by mast cells during activation? A. IL-1ß B. CCL2 C. IL-6 D. TNF-ɑ E. All of the above

E. All of the above

SARS-CoV2 causes hyperinflammatory response via what mechanism ? A. Activation of monocytes/macrophages B. Activation of dendritic cells C. Activation of mast cells D. Activation of T-cells E. All of the above

E. All of the above

A young-adult researcher named Andre recently returned to his home, which has two dogs, after a one year research project in the Arctic studying walruses. After he returned home, he started noticing a persistent fever with a temperature of 101.3F, muscle pain, and gastrointestinal distress. He made an appointment to see his PCP after he also developed splinter hemorrhages beneath his fingernails. His PCP decides to take a muscle biopsy, as she suspects the patient's symptoms are being caused by a parasite. What would be seen in a muscle biopsy specimen of this patient? A. Coiled encysted larvae in smooth muscle B. Adult worms in cardiac muscle C. Filariform larvae in skeletal muscle D. Eggs in smooth muscle E. Coiled encysted larvae in skeletal muscle

E. Coiled encysted larvae in skeletal muscle

A 57 year old male comes to the ER with abdominal cramps and constipation. He notices blood and pus in his stool. The bacteria stained pink on a gram stain and grew as yellow colonies when plated on MacConkey agar. The bacteria is also H2S negative. What best describes the virulence of the identified bacteria? A. The bacteria adheres to the intestinal epithelium through colonization factors and produces heat stable and heat labile enterotoxins. B. The bacteria adheres to the intestinal epithelium and destroys the microvilli. C. The B subunit of the toxin binds to 28S rRNA and inhibits protein synthesis. D. The bacteria invades the M cells in the Peyer's Patch and replicates in vacuoles. E. The bacteria invades the M cells in the Peyer's Patch and replicates in the host's cytoplasm

E. The bacteria invades the M cells in the Peyer's Patch and replicates in the host's cytoplasm

A 29 year old female received a blood transfusion during a busy night at the hospital and is now showing signs of yellowing of the skin and sclera of the eye, hypotension, and renal failure. What would be the primary cause of these symptoms? A. The patient has a decreased erythrocyte count. B. The patient has a build up of conjugated bilirubin. C. The patient has primary familial and congenital polycythemia D. The patient has physiologic jaundice. E. The patient was administered ABO-incompatible blood.

E. The patient was administered ABO-incompatible blood.

A group of medical students are reviewing a postmortem case of a 28-year-old female. She initially presented to the ER with a frontal headache, fever, nausea, and an altered sense of smell and taste. Her symptoms started about 5 days after returning from a hike where she went swimming in a hot spring. Her initial ER physical examination noted a positive Kernig sign and a stiff neck. The students recommended performing a brain tissue biopsy, and the results are shown below. Based on the biopsy findings, what organism is likely the causative agent? A. Acanthamoeba keratitis B. Balamuthia mandrillaris C. Leishmania donovani D. Trypanosoma cruzi E. Naegleria fowleri

E. Naegleria fowleri

A 21-year-old male by the name of Peter Parker accompanied his girlfriend to Africa to protect her as she foolhardily embarked upon a month-long journalism trip with no regard for her safety or health. His girlfriend (MJ) as usual got herself entangled with some dangerous criminals and contracted benign tertian malaria. Peter Parker, being a scientist himself, immediately diagnosed her based on her symptoms of a recurring pattern of chills, fever, and malarial rigors. What is the name for these set of symptoms? A. Hibby-Jibbies B. Febrile Periodicity C. Quotidian D. Exoerythrocytic Cycle E. Paroxysms

E. Paroxysms

Twenty-year-old Mary Jane Watson comes into the hospital with a paroxysms of fever, chills, and malarial rigors that appear generally every 48 hours or so, as best as her boyfriend can tell. She tells that she has just gotten back from Africa on a month-long journalism trip and the symptoms hit her just as she was getting home. Blood smears show the enlarged RBCs with Schüffner dots and golden-brown hemozoin pigment granules. The doctors also find that she is indeed positive for the Duffy blood group antigen on the RBC surface. What bacteria is MJ infected with? A. Babesia microti B. Leishmania columbiensis C. Leishmania peruviana D. Plasmodium falciparum E. Plasmodium vivax

E. Plasmodium vivax

A 20 year old male presents with fever, cough, and shortness of breath fourteen days after attending a house party. He admits to the attending in the emergency department that he knew he wasn't socially distancing and didn't wear a mask because he assumed that indulging himself in one night of fun wouldn't hurt anyone. He received a nasopharyngeal swab test and received his results 48 hours later. He tested positive for SARS-CoV-2. He was prescribed Remdesivir which is a RNA-dependent RNA polymerase inhibitor. What is the mechanism of action for this drug? A. Prevent synthesis of viral -ssRNA at the post viral entry stage. B. Prevent synthesis of viral dsRNA at the post viral entry stage. C. Prevent synthesis of RNA virus replication by inhibiting viral reverse transcriptase. D. Prevent synthesis of viral ssDNA at the post viral entry stage. E. Prevent synthesis of viral +ssRNA at the post viral entry stage.

E. Prevent synthesis of viral +ssRNA at the post viral entry stage.

In order to neutralize the acids created by bacteria in the large intestine, which of the following is necessary for this function ? A. Na+/glucose co-transporter B. bile salts C. Vitamin K D. Na+/H counter-transporter E. bicarbonate/chloride counter-transporter

E. bicarbonate/chloride counter-transporter

Proinsulin is converted into:

Insulin and C-peptide

What does high total bilirubin indicate?

Jaundice

What does high ALT indicate?

Liver damage (low -n/a)

What does high lactate dehydrogenase indicate?

MI, Pulmonary disease, Hepatic disease, hemolytic anemia, skeletal muscle disease, renal disease (low - n/a)

What does elevated eosinophils indicate?

Parasitic infection Allergic Helminthic infection

HDL indications

high density lipoprotein, good cholesterol; collects cholesterol from the body and transports to the liver

What does low erythrocyte sedimentation rate (ER) indicate?

Sickle cell anemia Spherocytosis polycythemia vera

What does low C-peptide indicate?

Type 1 diabetes

VLDL indicates

Very-low density lipoprotein; carries triglycerides from the liver to adipose tissue

What does low hematocrit indicate?

anemia bleeding bone marrow failure

What does low hemoglobin indicate?

anemia bleeding bone marrow failure

What does low HbA1c indicate?

anemia chronic blood loss

What does high RDW indicate?

anemias

A 21-year-old male presents to the Emergency Department complaining of fever, gastrointestinal distress, and muscle pain. A complete blood count is preformed, and the patient presents with marked eosinophilia. Upon taking the patients history the physician recalls that the patient has just moved out on his own after college and was recently experimenting with new pork dishes for dinner. What is the patient diagnosed with? What was the likely cause of infection? A. Ascaris lumbricoides; undercooked, dried, or smoked pork B.Trichinella spiralis; undercooked, dried, or smoked pork C. Enterobius vermicularis; undercooked beef D. Trichinella spiralis; seafood E. ETEC; Enterotoxigenic Escherichia coli

b) Trichinella spiralis; undercooked, dried, or smoked pork

What does high Alkaline phosphatase indicate?

bone or liver damage

What does low WBC (all) count indicate?

bone marrow failure

What does low reticulocyte count indicate?

bone marrow failure

What does low sodium indicate?

decreased intake diarrhea vomiting diuretics hyperglycemia HYPONATREMIA

What does low creatinine indicate?

decreased muscle mass

Amylase indications

detect and monitor pancreatitis

erythrocyte sedimentation rate (ESR) indication

detect inflammation

What does low glucagon indicate?

diabetes cystic fibrosis chronic pancreatitis pancreatic cancer

What does high glucagon indicate?

diabetes renal failure severe stress

What does high direct (conjugated) bilirubin indicate?

direct hyperbilirubinemia from gallstones, tumor, inflammation, scarring, or obstruction of the extrahepatic ducts

A 45-year-old female with a history of left toe amputation, uncontrolled diabetes, and left eye blindness presents to the emergency department complaining of right upper quadrant abdominal pain. Lab work performed in the ED showed elevated ALT and AST. Gastroenterology and general surgery were consulted on the case prior to general surgery taking lead and opting for surgical intervention of suspected gallstones following a positive ultrasound. Following initial incisions of the patient's abdominal wall, "knobby coated" oval eggs were observed by operating room staff. Which of the following organs is NOT affected during the migration of this organism? a) Lung b) Liver c) Small Intestine d) Heart e) Spleen

e) Spleen

What does high potassium indicate?

excessive intake renal failure hemolysis acidosis dehydration HYPERKALEMIA

What does high HDL indicate?

familial HDL excessive exercise

What does high LDL or VLDL indicate?

familial LDL familial hypercholesteremia glycogen storage disease alcohol

What does high triglycerides indicate?

glycogen storage disease poorly controlled diabetes renal failure

What does low platelet count indicate?

hemorrhage

what does high reticulocyte count indicate?

hemorrhage hemolytic anemias

Where is direct bilirubin found?

in the liver

What does high blood glucose (fasting or casual) indicate?

hyperglycemia diabetes

what does high urine glucose indicate?

hyperglycemia diabetes (low -n/a - remember that it should be completely reabsorbed at kidneys - normal = none in urine)

What does high glucose tolerance indicate?

hyperglycemia diabetes gestational diabetes

What does high postprandial glucose indicate?

hyperglycemia diabetes gestational diabetes

What does low blood glucose (fasting or casual) indicate?

hypoglycemia insulin overdose

What does low postprandial glucose indicate?

hypoglycemia insulin overdose

Where is indirect bilirubin found?

in the spleen and RBCs

What does high Hemoglobin indicate?

increased RBCs polycythemia vera dehydration

What does high RBC count indicate?

increased RBCs polycythemia vera dehydration hemoglobinopathies

What does high sodium indicate?

increased intake dehydration HYPERNATREMIA

C reactive protein (CRP) indications

indicates inflammation (non-specific)

What does high WBC (all) count indicate?

infection leukemias inflammation dehydration

What does high erythrocyte sedimentation rate (ER) indicate?

inflammation cancer severe anemias

What does high C reactive protein (CRP) indicate?

inflammation (non-specific) (low - n/a)

Lactate dehydrogenase indications

intracellular enzyme indicative of injury or disease involving the heart, liver, red blood cells (RBCs), kidneys, skeletal muscle, brain, and lungs

What does low Alkaline phosphatase indicate?

low phosphate, malnutrition, scurvy, milk-alkali syndrome

What does high MCV -or- high MCH indicate?

macrocytic anemias

What does low triglycerides indicate?

malnutrition hyperthyroidism

Glucagon indications

measure glucagon in the blood

Triglycerides indications

measurement of cholesterol and lipoproteins (measurement of fats in circulation)

Glucose postprandial indications

measures blood glucose 2 hours after a meal - diagnose diabetes

What does low HDL indicate?

metabolic syndrome

What does low MCV -or- low MCH indicate?

microcytic anemias

Platelet count indication

monitor bleeding/clotting

HbA1c diabetic control values:

nondiabetic: 4-5.9% good diabetic control: <7% fair diabetic control: 8-9% poor diabetic control: >9%

What does normocytic MCV mean?

normal RBC size

What does high amylase indicate?

pancreatitis, salivary gland or gallbladder disease, kidney failure (Low - n/a)

Sodium indications

part of basic electrolyte panel: used to check renal function and for dehydration

Potassium indications

part of basic electrolyte panel: used to heart and kidney function

What does high hematocrit indicate?

polycythemia vera increased RBCs dehydration

what does high HbA1c indicate?

poorly controlled diabetes

what does high MCHC indicate?

spherocytosis

Urine glucose indications

screen for diabetes, can also be a part of urinalysis

What does low MCHC indicate?

thalassemia

What does high platelet count indicate?

thrombocytosis polycythemia vera

Alkaline Phosphatase (ALP) indication

used to detect disease of the liver and bone

Blood glucose indications

used to evaluate diabetic patients

C- peptide indication

used to evaluate diabetic patients to monitor insulinomas to determine the difference between Type 1 and Type 2 diabetes - fasting test - 1 hour after glucose

Alanine Aminotransferase (ALT, SGPT) indications

used to identify liver damage

What does elevated lymphocytes indicate?

viral infection

What does low aspartate aminotransferase (AST) indicate?

- Acute renal disease - Beriberi - Diabetic ketoacidosis - Pregnancy - Chronic renal dialysis

What are the components of CBC and differential?

- RBC - Hemoglobin - Hematocrit, packed red blood cell volume - MCV (Mean corpuscular value = hematocrit(%) x 10/RBC) - MCH (mean corpuscular hemoglobin = hemoglobin x 10/RBC) - MCHC (Mean hemoglobin corpuscular hemoglobin concentration = hemoglobin x 100/hematocrit) - RDW ( Red cell distribution width) - Reticulocyte count

glycosylates hemoglobin A1C (HbA1c) indications

- diagnose and monitor diabetes - long-term index of the patient's average blood glucose levels

Anti-viral medications, such as Hydroxy-Chloroquine and Chloroquine, are believed to be most effective during which time period, also known as the "Golden Window"? A. within 48 hours of the onset of symptoms B. within 72 hours of the onset of symptoms C. within 48 hours of admission to the hospital D. within 72 hours of admission to the hospital

A. within 48 hours of the onset of symptoms

A 43-year-old female presents to her local clinic with fatigue, shortness of breath, pale skin, and abdominal pain. The patient notifies her attending physician that she has had episodes of bright red blood appearing in her urine, at night. The physician orders a flow cytometry test, which reveals an abundance of red blood cells that are deficient in glycosylphosphatidylinositol-linked membrane proteins. Which of the following best explains the reason for the lack of such proteins? A. A Random inactivation of the X-linked PIGA gene B. Deletion of two 𝛂-globin genes C. Antibodies that recognize and lead to RBC destruction D. Cytoskeleton defects E. Abnormalities in glutathione metabolism

A. A Random inactivation of the X-linked PIGA gene

A 28-year-old female presents to the ED complaining of profuse watery diarrhea (rice water stools), fatigue, and increased thirst. The patient reveals she went on a trip to Bangladesh to visit her grandmother and returned back to the United States, 2 days ago. The ER physician runs a series of lab tests and the results revealed the patient's symptoms are consistent with Vibrio Cholerae. The patient's symptoms are caused by the cholera toxin. What is the mode of action of the cholera toxin? A. Activation of adenylate cyclase, increasing intracellular cAMP B. Activation of guanylate cyclase, increasing cGMP C. Protein synthesis inhibition D. Ribosylation of small GTPases (Rho) E. M cell invasion

A. Activation of adenylate cyclase, increasing intracellular cAMP

A 32 year old male visits his doctor's office after returning from a trip abroad. He tells his doctor that he has had persistent diarrhea since his return and notes eating raw seafood while abroad. The doctor takes a stool sample and finds a large number of parasites that are usually associated with being nonpathogenic. Which organism did the doctor find and what would be prescribe to treat his patient? A. Blastocystis and metronidazole B. E. Polecki and metronidazole C. Blastocystis and vancomycin D. E. Nana and erythromycin E. E. Histolytica and iodoquinol

A. Blastocystis and metronidazole

An 84-year-old man presents to the ED with cramps, a fever, and watery diarrhea with a foul smell. The patient mentions that he was recently prescribed antibiotics for a skin infection and is almost finished with the course. Lab findings revealed leukocytes and occult blood in his stool, indicative of pseudomembranous colitis. What is the bacteria that is responsible for causing this patient's pseudomembranous colitis? A. C. difficile B. Y. pestis C. EIEC D. Shigella Salmonella

A. C. difficile

A 21-year-old male presented to his PCP with complaints of itchy, erythematous skin rash in bilateral lower extremities that has been ongoing for the past month. He did travel to Palm Beach, Florida to enjoy the scenery and the walks on the beach. Upon physical examination, skin examination revealed cutaneous, serpentine-like, skin lesions in the medial aspect of bilateral lower extremities. CBC revealed elevated eosinophils. Based on the findings, what is the typical host of this organism? A. Cats and Dogs B. Humans C. Racoons D. Copepods E. Swine

A. Cats and Dogs

A 70-year-old woman presents to her clinic with watery diarrhea and abdominal cramps for the past week. She was hospitalized few weeks ago for pneumonia and treated with penicillin. Physical examination reveals lower abdominal tenderness. The initial laboratory evaluation of the stool reveals the presence of leukocytes. What is the most likely causative agent and the virulence factor(s) responsible for the symptoms? A. Clostridium Difficile -toxin A (enterotoxin) and toxin B (cytotoxin) B. Shigella - Shiga toxins (Stx1 and Stx2) C. Clostridium Difficile - Heat labile toxin (LT-I or LT-II) D. EHEC - Heat labile toxin (LT-I or LT-II) Y. enterocolitica- M cell invasion

A. Clostridium Difficile -toxin A (enterotoxin) and toxin B (cytotoxin)

A patient returns from a tropical vacation still experiencing Diarrhea, bloody stool, fever and abdominal pain. The cause of this particular infection is determined to be a protozoan. What is the method of transmission of the protozoan infection? A. Consuming fecally contaminated water B. Bite from sand fly C. Contact with the saliva of an infected animal D. Swimming in contaminated water E. Bite from the reduviid bug

A. Consuming fecally contaminated water

A patient who just recently gave birth to her baby complains that her newborn has failed to have a bowel movement within 48 hours after birth. Assessing the newborn's belly, distension of the baby's abdomen to a diameter of 3-4 inches was noticed. It was determined that the accumulation of a tremendous quantity of fecal matter in the colon led to this distension. Which of the following would be the most likely cause of this disease? A. Deficiency of ganglion cells of the colon B. Helicobacter pylori C. Gluten D. Allergic or immune destructive effect E. Excessive stimulation of the parasympathetic nervous system

A. Deficiency of ganglion cells of the colon

A mother brings her 3 year old child to the pediatrician with a chief complaint of pruritus and loss of sleep. She informs the pediatrician that she recently enrolled her child in day care and that many of the other children share the same symptoms. The pediatrician diagnosed the child with Enterobius vermicularis and treated them accordingly. However, the child comes back 2 days later but now with complaints of abdominal pain, flatulence and intermittent diarrhea. This time the pediatrician treats the child with doxycycline and alleviates his symptoms. What is the responsible organism for the second set of symptoms? A. Dientamoeba fragilis B. Giardia duodenalis C. Entamoeba histolytica D. Clostridium difficile E. Necator Americanus

A. Dientamoeba fragilis

A patient presents with intermittent diarrhea and abdominal pain in the ER. Upon taking her history she indicates she has been out of the country in the last month to many different areas of the world. The stool of the patient is analyzed and reports findings of small trophozoites containing 1-2 nuclei. The pathogen is identified as an ameba but the structure of the trophozoite is like a flagellate. The patient is prescribed an antimicrobial agent. What is the pathogen in question? A. Dientamoeba fragilis B. Cryptosporidium C. Balantidium Coli D. Cystoisospora E. Trichomonas vaginalis

A. Dientamoeba fragilis

A five year old boy accompanied by his mother arrives at his pediatrician's office for an early morning appointment. The boy mentions he's been having difficulty sleeping through the night due to severe pruritus by his rectum. The patient denies wearing any new clothes or using new lotions and soaps that could have caused his itching. The pediatrician orders an anal swab which was later examined and showed thin walled, ovoid eggs. Which organism is responsible for this patient's symptoms? A. Enterobius vermicularis B. Ascaris lumbricoides C. Trichuris trichiura D. Strongyloides stercoralis E. Ancylostoma duodenale

A. Enterobius vermicularis

A 6 year old child is sent home from daycare due to a fever and watery diarrhea. After two days the child's mother noticed the watery diarrhea had progressed to bloody diarrhea and decided to take the child to the doctor. The doctor performed tests to differentiate Shigella from Enteroinvasive E. coli, prior to diagnosing the child with Shigellosis. What is one characteristic that is different between Shigella and Enteroinvasive E. coli? A. Enteroinvase E. coli does not produce toxins B. Shigella does not produce toxins C. Enteroinvasive E. coli is only spread through person to person contact D. Enteroinvasive E. coli does not progress into bloody diarrhea E. Shigella is highly resistant to gastric acid

A. Enteroinvase E. coli does not produce toxins

A 5 year old male patient was brought to the clinic because his parents are concerned about bloody diarrhea. The patient was previously healthy but started developing symptoms after attending a birthday party at a petting zoo. Lab test revealed that the microbe responsible for the GI infection is a gram-negative rod that belongs to the Enterobacteriaceae family. Which of the following lab results are expected for bacteria of this family? A. Glucose fermenter, Catalase +, oxidase - B. Glucose fermenter, Catalase -, oxidase + C. Glucose non-fermenter, Catalase +, oxidase + D. Glucose fermenter, Catalase -, oxidase - E. Glucose non-fermenter, Catalase +, oxidase -

A. Glucose fermenter, Catalase +, oxidase -

A 39-year-old male of Mediterranean descent presents to his physician with complaints of fatigue, pallor, and shortness of breath. The patient says that he is currently taking aspirin for his heart condition and did not eat anything out of the ordinary except for some fava beans in a meal a few days ago. The physician orders a laboratory test to confirm his suspicion and diagnoses the patient with hemolytic anemia. Which enzyme is deficient in this patient and what can be seen on a blood smear because of this? A. Glucose-6-Phosphate Dehydrogenase, Heinz bodies B. Glucose-6-Phosphate Dehydrogenase, Negri bodies C. Pyruvate Dehydrogenase, Guarneri bodies D. Glycogen Phosphorylase, Heinz bodies E. Pyruvate Dehydrogenase, Negri Bodies

A. Glucose-6-Phosphate Dehydrogenase, Heinz bodies

A 24-year-old woman presents to her physician with complaints of fatigue and weakness for several months. She mentioned how lately she has had trouble walking up the stairs to her apartment on the third floor, which she used to be able to do easily. Her partner also noticed that lately she's been very pale. On physical examination the physician noticed conjunctival pallor, but an otherwise healthy appearance. Lab results would reveal a decrease in what type of hemoglobin? A. HbA (α2β2) B. HbA2 (α2δ2) C. HbA (α2β1) D. HbF (α2γ2) E. HbA2 (α2β2)

A. HbA (α2β2)

. A 21-year old female presents to her university's Student Health Center with unusual tiredness and pale skin. She claims that she has been sleeping regularly and maintaining a healthy diet. Her PA believes that she may have iron deficiency and suggests taking an iron supplement. However, she returns to the Student Health Center within a month expressing the same concern. After completing a routine blood test, her PA discovers hypochromia, microcytosis, basophilic stippling, and target cells in the patient's peripheral blood smear. She has reason to believe that the patient has 𝛃-Thalassemia minor. Which of the following is diagnostically useful? A. Hemoglobin electrophoresis revealing increased HbA2 levels B. Lab test revealing a normal total iron-binding capacity C. Genetic screening indicating 𝛃-globin tetramer, HbH D. Direct Coombs antiglobulin test indicating agglutination E. Genetic screening indicating a PIGA mutation

A. Hemoglobin electrophoresis revealing increased HbA2 levels

A 25 year old female presents to the clinic with anemia, splenomegaly, and jaundice. She has an inherited disorder that reduces the assembly of the red cell skeleton and destabilizes the plasma membrane. Which of the following disorders does she present with? A. Hereditary Spherocytosis B. Paroxysmal Nocturnal Hemoglobinuria C. Immunohemolytic Anemia D. Thalassemia E. Sickle Cell Disease

A. Hereditary Spherocytosis

Ascaris lumbricoides is a tropical parasite with a complex life cycle. Once in the body, what is the proper route of maturation for these nematodes in humans? A. Ingestion of infective egg -> Migration of larva to liver, heart, then lungs -> larvae grow and molt for ~3 weeks before they are coughed up then swallowed -> Maturation into adult worms in the small intestine B. Ingestion of infective egg -> Larvae hatch in small intestine then migrate to the large intestine -> Maturation into an adult in 2-6 weeks C. Infective larvae penetrate the skin and enter circulation -> Migration to the lungs before being coughed up and swallowed -> Maturation into adults in the small intestine D. Infective larvae introduced by a mosquito -> Migration to lymphatic system where they mature into adults E. They are not able to undergo their complete life cycle in humans

A. Ingestion of infective egg -> Migration of larva to liver, heart, then lungs -> larvae grow and molt for ~3 weeks before they are coughed up then swallowed -> Maturation into adult worms in the small intestine

A 27-year-old male from Senegal presents to the clinic with complaints of occasional fatigue but no other symptoms. Lab results show increased MCHC with intracellular crystals seen on blood smear. Molecular analysis reveals the patient is homozygous for HbC mutation. What are the effects of this hemoglobin variant? A. It promotes water loss from the cell by activating K+ transporter and lowers the hemoglobin solubility by replacing glutamate to lysine, resulting in the precipitation of the hemoglobin variant. B. It inhibits water loss from the cell by activating K+ transporter and lowers the hemoglobin solubility by replacing glutamate to lysine, resulting in the precipitation of the hemoglobin variant. C. It inhibits water loss from the cell by activating K+ transporter and increases the hemoglobin solubility by replacing glutamate to lysine, resulting in the precipitation of the hemoglobin variant. D. It promotes water loss from the cell by activating K+ transporter and increases the hemoglobin solubility by replacing glutamate to lysine, resulting in the precipitation of the hemoglobin variant. E. It promotes water loss from the cell by activating K+ transporter and lowers the hemoglobin solubility by replacing glutamate to glycine, resulting in the precipitation of the hemoglobin variant.

A. It promotes water loss from the cell by activating K+ transporter and lowers the hemoglobin solubility by replacing glutamate to lysine, resulting in the precipitation of the hemoglobin variant.

What test is first used to differentiate COVID-19 from the flu and pneumonia? A. Nasopharyngeal Swab B. qPRC C. Nucleic Acid Test D. Complete Blood Count E> X-Ray of the Lungs

A. Nasopharyngeal Swab

Why does this study look at temperature and humidity as possible causes of reproductive variability? A. Other influenza viruses have shown improved reproductive rates in dry, cooler air B. Other outbreaks of SARS-CoV have shown improved reproductive rates in high humidity and warmer temperatures C. More data can be collected from areas with dry, cooler air D. More data can be collected from areas with higher humidity and warmer temperatures

A. Other influenza viruses have shown improved reproductive rates in dry, cooler air

A healthy 25 year old male ate a big hamburger for dinner and his body is in the process of digesting the fats he has eaten. What enzyme is responsible for the majority of triglyceride digestion from this meal? A. Pancreatic Lipase B. Lecithin C. Bile Salts D. Pepsin E. Maltase

A. Pancreatic Lipase

Which of the following cells were isolated and used to explore cellular immune responses to SARS-CoV-2? A. Peripheral blood mononuclear cells (PBMC) B. Granulocytes C. Basophils D. Erythrocytes

A. Peripheral blood mononuclear cells (PBMC)

A 55 y/o female presents to the ER with shortness of breath, fever, cough, and loss of smell. Her temperature is 100.7 degrees, pulse 110/min, and oxygen saturation 90%. The patient is admitted into the hospital and subsequently diagnosed with SARS-CoV-2. After 10 days, the patient is chosen to be part of a double-blind, randomized, placebo-controlled drug trial and treated with an experimental drug Remdesivir. According to Article #6, what is Remdesivir known to target? A. RNA dependent RNA polymerase B. Ion channels C. DNA polymerase D. ACE2 receptor

A. RNA dependent RNA polymerase

A patient presents to an Urgent Care Clinic with extreme diarrhea. The physician there is aware that whenever significant amounts of intestinal secretions are lost to the exterior the sodium reserves of the body can sometimes be depleted to lethal levels within hours. Normally, however, less than 0.5% of the intestinal sodium is lost in the feces each day. Most likely, why is this? A. Rapid absorption of sodium through the intestinal mucosa B. Reabsorption of sodium in the renal system C. Excessive intake of sodium on a daily basis D. Sodium cotransporter between the ECF and ICF E. Isosmotic absorption of water

A. Rapid absorption of sodium through the intestinal mucosa

Oliver sat down in front of his students as he recalled a story from a village he once visited in Africa. A man in the village infected with a pathogen showed a maculopapular rash with severe pruritus that had been present for multiple years. His skin showed multiple thick areas with pigmentation. After going to an advanced hospital outside his village, skin snips were taken and a larva transmitted by the Simulium (blackfly) vector was revealed. What is one major symptom that was left out of the story that is also seen in this infection? A. River Blindness B. Toxic Megacolon C. Winterbottom's sign D. Swimmer's itch E. Hepatosplenomegaly

A. River Blindness

Why is the receptor binding domain of spike protein (S-RBD) for SARS-CoV-2 a candidate for a target in the SARS-CoV-2 vaccine? A. S-RBD induces a broad T-cell immune response B. S-RBD induced production of only CD4 C. S-RBD-specific antibodies were detected in recovered patients D. S-RBD induced production of only CD8

A. S-RBD induces a broad T-cell immune response

Based on the image in Figure 1, what protein, when, mutated, could result in the loss of attachment and therefore entry of Coronavirus into the host cell? A. S-protein B. N-protein C. M-protein D. C-reactive protein E. Envelope protein

A. S-protein

. A 19-year-old patient presents to the ED with cramps, vomiting, nausea, a low-grade fever, and watery, non-bloody diarrhea two days after returning home from a trip to Belize. PCR analysis of the fecal specimen suggests that the patient's diarrhea is due to inhibition of fluid absorption and an increase in chloride secretion in the small intestinal epithelium due to infection with enterotoxigenic E. coli (ETEC). The patient's diarrhea is caused by which of the following? A. STa enterotoxin-mediated increase in cGMP as well as LT-I mediated increase in cAMP. B. STb enterotoxin-mediated increase in cGMP as well as LT-II mediated increase in cAMP. C. STa enterotoxin-mediated increase in cAMP as well as LT-I mediated increase in cGMP. D. STb enterotoxin-mediated increase in cGMP as well as LT-II mediated increase in cGMP. E. LT-I enterotoxin-mediated increase in cAMP.

A. STa enterotoxin-mediated increase in cGMP as well as LT-I mediated increase in cAMP.

A fifteen-year-old female is brought into the emergency room with symptoms of vomiting, abdominal pain and bloody diarrhea, but is afebrile. She mentions that she recently consumed tacos containing ground beef a few days prior but notes that the ground beef may have been slightly undercooked. Which of the following is most likely the pathogenesis of the organism that is causing her symptoms? A. Shiga Toxins (Stx1 and Stx2) that cleave 28s rRNA to disrupt protein synthesis B. Heat-stable ST and heat-labile LT toxins that stimulate hypersecretion of fluids by respectively increasing cGMP and cAMP C. Destruction of the microvillus with attachment/effacement histopathology D. Bacterial invasion and destruction of colonic epithelium E. Autoagglutination in a "stacked-brick" arrangement over small intestine epithelium

A. Shiga Toxins (Stx1 and Stx2) that cleave 28s rRNA to disrupt protein synthesis

The ER admits a 7 year old male with a suspected gastrointestinal obstruction. His patient history shows 6 admissions within the past year for repeated bacterial infections, 3 of which resulted in ICU care. His mother also reports he has frequent skin infections and diarrhea. Family history documents the mother as a carrier of the CYBB gene mutation affecting neutrophil function. What is the most probable defect in neutrophil function causing the boy's symptoms? A. The NADPH oxidase system is unable to form cytochrome B558 on the phagolysosome plasma membrane B. The myeloperoxidase (MPO) system of is unable to produce hypochlorous acid C. The NADPH oxidase system is unable to activate Rac-2 GTPase D. Defensins cannot produce the antimicrobial peptide cathelicidins E. The myeloperoxidase (MPO) system is unable to convert reactive nitrogen intermediates (RNIs)

A. The NADPH oxidase system is unable to form cytochrome B558 on the phagolysosome plasma membrane

21M presents to his PCP with complaints of constipation. He also reports not urinating very much, and when he does it is dark yellow. He is extremely active and plays club soccer, football, and basketball every week. He admits that he often forgets to drink water while playing. He is otherwise healthy. Which characteristic of the large intestine is most responsible for allowing more complete absorption of electrolytes and water than the small intestine? A. Tighter tight junctions B. Presence of gut flora C. It is longer, which allows more time for absorption D. More villi E. Secretion of bicarbonate ion

A. Tighter tight junctions

A 23 y.o. Female presents to her PCP with scant watery vaginal discharge, itching, burning, and painful urination for the past week. She reports being sexually active with her boyfriend. Microscopic examination of the vaginal discharge will most likely reveal which of the following: A. Trophozoites with flagella and short undulating membrane B. Cysts and trophozoites C. Small trophozoites with one or two nuclei and karyosome consisting of granules D. Large trophozoites with cilia and macronucleus and cysts E. Oocysts

A. Trophozoites with flagella and short undulating membrane

After a full-term delivery, a mother notices a slight yellowing of her baby's skin. Test results come back which show high levels of bilirubin. The doctor orders phototherapy and the jaundice improves. What type of bilirubin was causing the symptoms and why was phototherapy effective? A. Unconjugated, chemical changes increasing the water solubility of bilirubin B. Unconjugated, chemical changes decreasing the water solubility of bilirubin C. Conjugated, chemical changes increasing the water solubility of bilirubin D. Conjugated, chemical changes decreasing the water solubility of bilirubin E. Conjugated, no change in solubility of bilirubin

A. Unconjugated, chemical changes increasing the water solubility of bilirubin

Why was a wild type mouse not used to test the effects of RDV? A. Wild-type mice produce a serum esterase absent in humans, that decreases the half- life of RDV. B. Wild-type mice are unable to metabolize RDV to active TP. C. Wild-type mice produce a serum esterase absent in humans, that increases the half- life of RDV. D. Wild-type mice quickly metabolize RDV to active TP.

A. Wild-type mice produce a serum esterase absent in humans, that decreases the half- life of RDV.

A researcher is studying the effects of pepsin, which is expressed as pepsinogen by gastric chief cells, an important protein digesting enzyme in the stomach. After genetically modifying the chief cells to inhibit secretion of pepsinogen in a population of lab rats, their goal is to study the impact this inhibition has on pepsin in relation to protein digestion. Thus, which of the following macromolecules is most likely no longer hydrolytically cleaved due to the absence of pepsin? A. collagen B. keratin C. elastin D. fibronectin E. myoglobin

A. collagen

What factor suggests that the host immune system is involved in pathogenesis in a patient suffering from a severe case of Sars-Cov-2? A. higher plasma levels of pro-inflammatory cytokines B. Defective Type I and Type II interferons C. Human AB serum D. Escherichia coli transformed with pET28- N-6xHis E. Immune system is not really involved

A. higher plasma levels of pro-inflammatory cytokines

A 28 years old female presents to the Emergency Room with complaints of pain with urination accompanied with bloody urine. She recently studied abroad in Africa and while she was there, she did develop acute dermatitis while she was there, but she has been back in the US for a month now she didn't think it was important. The physician suspects she is suffering from a parasitic Schistosoma haematobium infection. What test would the physician use to confirm the diagnosis? A. presence of eggs in a urine sample B. Presence of eggs in a blood smear C. presence of eggs in the stool D. CT scan E. tape test

A. presence of eggs in a urine sample

A 25 year old graduate student researcher is interested in the development of Erythrocytes. He exposes common myeloid progenitor cells (CMP) to erythropoietin, IL-3, and IL-4. He then decides to block expression of transcription factor GATA-1. What is the first precursor cell in erythropoiesis that the researcher is preventing formation of? A. proerythroblast B. basophilic erythroblast C. myeloblasts D. megakaryoblast E. basophil

A. proerythroblast

Describe the pathogenesis of encapsulated Enterobacteriaceae? A. protection from phagocytosis B. expression of alternating antigens C. transfer of bacterial virulence factors into targeted eukaryotic cell D. lipid A component of LPS E. produce siderophores

A. protection from phagocytosis

A 4-year-old male is brought to his pediatric ophthalmologist when his mother notices the child complaining of light sensitivity, blurry vision and frequently squinting to favor one eye. The mother is worried as her son already struggles with hearing loss and seizures. Upon examination, the ophthalmologist notes inflammation and retinal scarring suggestive of chorioretinitis; it is suspected that the infection was originally acquired transplacentally. Histological examination of a tissue sample shows the formation of a cyst with trophozoites. What is the essential reservoir for the suspected organism? A. Birds B. Felines C. Humans D. Mosquitos E. Bats

B. Felines

A 28 year old female presents to her Primary Care Physician for her annual physical. Upon talking with the patient, the physician finds it necessary to draw a blood test in order to determine levels of the patient's bodily iron stores. When the physician obtains the results of the blood tests, they determine that the levels of bodily iron stores are increased. What protein was elevated in the patient's blood that most likely led to this determination? A. Amylase B. Ferritin C. Hemoglobin D. Albumin E. Transferrin

B. Ferritin

A 36-year-old female goes to her General Family Physician after complaining of feeling tired and short of breath during the week. She complains of difficulty concentrating and frequent headaches and having a hard time falling asleep although she cuts caffeine in her daily intake. Upon physical examination, the doctor noticed that the patient's skin was pale and so the doctor ordered for blood to be drawn and sent to the laboratory for CBC. Once the results arrived, it illustrated the patient has a low hemoglobin concentration of 10.3g/dL and further laboratory results showed that the patient had a low vitamin B12 as well. The result illustrates ... A. Sickle Cell disease B. Anemia C. Barr body D. Neutrophilic E. Leukocytes

B. Anemia

An 11 month old child presents to the clinic after the rapid development of watery diarrhea accompanied by fever and vomiting starting within hours after arriving home from daycare yesterday. The parents deny any blood in the patient's stool and also say they have not fed him any undercooked meat. It is determined that the illness was spread via fecal oral from another child attending daycare. After lab tests confirm the doctors diagnosis, it is known that this specific strain possesses a cluster of virulence genes located on a locus of enterocyte effacement (LEE). What is the likely mechanism this bacteria uses to initiate infection causing watery diarrhea? A. Shiga toxins (Stx1 and Stx2) B. Attachment to epithelial cells leading to effacement of microvilli (A/E histopathology) C. Heat stable toxin (STa or STb) causing an increase in cAMP D. Invasion and destruction of colonic epithelium E. Heat labile toxin (LT-I or LT-II)

B. Attachment to epithelial cells leading to effacement of microvilli (A/E histopathology)

A 30-year-old male presents to the emergency department with complaints of fatigue, generalized weakness, and shortness of breath. He recently started seeing a new primary care physician after moving to the area. He was diagnosed with sickle cell anemia as a child but has not had proper follow up since graduating from college. His prior diagnosis is likely causing his shortness of breath due to which of the following? A. Lack of spectrin proteins B. Inability of RBCs to maintain shape for proper oxygen transport C. Antibodies attacking RBCs D. Damaged RBCs blocking the airway E. Defective erythropoietin receptors

B. Inability of RBCs to maintain shape for proper oxygen transport

A family in the United States adopted a child from Africa at three months old. Now, three months later, the child is six months old, the family notices that their growth has stopped and their cheekbones have become large and malformed. A CBC shows that their HbF is elevated and the child is showing signs of hepatosplenomegaly. It is deduced that the child has some severe mutation/possible deletion in the gene of the synthesis of globin chains. What chains would be targeted in this condition? A. Alpha B. Beta C. Gamma D. Delta E. Zeta

B. Beta

A 67 year old female with rheumatoid arthritis presents to the clinic with hemiparesis, visual impairment, confusion and lethargy. The patient has been treating her RA with immunosuppressants for 5 years. The patient lives alone with 2 cats and recently returned from a week long trip to the Oregon coast. Which form of the infectious agent is responsible for her presentation? A. Tachyzoite B. Bradyzoite C. Pseudopodium D. Immature oocyst E. Pyriform bodies

B. Bradyzoite

A 23 year old male presents to his physician with complaints of fever, chills, headaches and vomiting after returning from travel in West Africa. The patient admits that he did not receive any prophylactics or immunizations before his trip, as he "didn't know he was supposed to." What type of treatment should the patient receive for his illness? A. Diphenoxylate and loperamide B. Chloroquine C. Azithromycin D. Metronidazole E. No treatment needed- self limiting

B. Chloroquine

A patient and his sister returned home from a trip to Nantucket Island 2 weeks ago. They recount hiking and spending time along the trails on the island. Both the patient and his sister had been feeling "under the weather" for the past 5 days, before developing fevers. The patient recalls feeling worse than his sister yesterday; he noticed that his face looked very pale in the mirror and he felt extremely fatigued. Bloodwork for the patient revealed an elevated reticulocyte count while the sister's bloodwork revealed a normal reticulocyte count. Blood smears of both the patient and his sister were examined. The sister's blood smear appeared to be negative while the patient's blood smear revealed ring forms, tetrads and paired merozoites. What would be the best treatment for this patient? A. Suramin and Melarsoprol B. Clindamycin and Quinine C. Chloroquine and Primaquine D. Pyrimethamine and Sulfadiazine E. Pentamidine and Ketoconazole

B. Clindamycin and Quinine

A 50 year old man returns from a business trip to a developing country, where bacterial diarrheal diseases are common. Upon returning to the United States, the man starts feeling symptoms of watery diarrhea and abdominal cramps 1-2 days later. The man is informed by his PCP that he has an Enterotoxigenic E. coli infection and most likely contracted it from his recent travel. How is this bacterial infection primarily acquired? A. Person-to-person spread B. Consumption of fecally contaminated food or water C. Contact with bodily fluids D. Through aerosols E. Through contact with an animal vector

B. Consumption of fecally contaminated food or water

A 50-year old man with a history of heavy alcohol use has presented to the Emergency Department citing increased thirst, nausea, and fatigue. Upon further inspection, it is found that he has cirrhosis of the liver as a result of secondary hemochromatosis. Which of the following changes inside of his body can likely be attributed to this condition? A. Decreased erythroferrone B. Decreased hepcidin C. Decreased RBC progenitors D. Decreased iron absorption in the gut E. Increased RBC production

B. Decreased hepcidin

A 22 year old female begins working on a fruit farm where she decided to eat an unwashed peach. She later learned that she consumed Ascaris lumbricoides eggs on the peach that grew from contaminated soil. What would need to be detected or what diagnostic test was used in order to arrive at this definite conclusion? A. Colorless growth on Sorbitol-MacConkey Agar B. Detection of eggs in stool sample C. CBC D. Thiosulfate citrate bile salts sucrose agar E. Blood smear

B. Detection of eggs in stool sample

A 24-year-old male presents to the clinic with complaints of extreme fatigue, weakness and pale skin appearance. Lab evaluation including CBC, reticulocyte count, ferritin test and peripheral blood smear indicate a normal RBC count with abnormally low ferritin in the blood. The red blood cells also appear to be small and pale indicating microcytic and hypochromic anemia. What is the cause of the microcytic red blood cells? A. Patient has a folate or vitamin B12 deficiency B. Developing red blood cells continue to divide past their normal stopping point C. Patient has Epo receptor mutation D. Patient has a vitamin B6 deficiency E. Differentiating hematopoietic cells remain in an immature and proliferative state

B. Developing red blood cells continue to divide past their normal stopping point

What virus was the drug, Remdesivir (RDV) evaluated for the treatment of before the pandemic? A. Zika virus B. Ebola virus C. Influenza virus D. Hepatitis B virus

B. Ebola virus

Alexa, A 19-year-old female of Mediterranean decent seeks advice from her doctor after feeling abnormally fatigued. Upon physical examination, the doctor notices Alexa is more pale than usual. Previously, she had not sought medical attention because she had attributed her fatigue to working tirelessly to become Valedictorian in high school. Before high school, Alexa claims she was asymptomatic. Her doctor orders a CBC, hemoglobin electrophoresis, and molecular genetic testing. Lab results find that she has microcytic, hypochromatic RBCs due to decreased b-chain synthesis. What other lab result can be attributed to why Alexa was previously asymptomatic? A. Decreased HbF B. Elevated HbF C. Excess b-chains D. d0 /b0 - heterozygous E. b0 /b0 homozygous

B. Elevated HbF

A 30 y.o african male presents to his primary care physician with a fever, dark urine, fatigue, and pale skin. The patient explains to his physician that he started taking chloroquine, an antimalarial medication, in preparation for his trip to Asia, and it was shortly after that he started experiencing these symptoms. Physical examination is remarkable for splenomegaly. The clinical workup shows evidence of hemolysis, increased bilirubin, and anemia. Blood smear findings include darkly stained regions within the red blood cells also known as Heinz bodies. The physician suspects that his symptoms are being caused by a mutation in a gene. What is the most likely diagnosis in the patient? A. Niacin deficiency B. Glucose 6 phosphate dehydrogenase deficiency C. B12 deficiency D. Thiamine deficiency E. Increased generation of glutathione

B. Glucose 6 phosphate dehydrogenase deficiency

A 30 year old female presents to her family physician with cramps, fever, nausea, and complaints of watery and bloody diarrhea. The physician notes that the patient also decided to self-medicate with leftover antibiotics right after she returned from goat yoga with her friends. After immediately being referred to the ER for dialysis, later lab tests show presence of oxidase negative, indole positive, lactose fermenting, Sorbitol negative organism in her stool. Which of the following best describes the bacteria's classification and structure? A. Gram positive bacteria, thick peptidoglycan layer with teichoic acid B. Gram negative bacteria, thin peptidoglycan layer with lipid A component C. Gram positive bacteria, thick peptidoglycan layer with LPS D. Gram negative bacteria, thin peptidoglycan layer with teichoic acid E. Gram negative bacteria, thick peptidoglycan layer with H antigen

B. Gram negative bacteria, thin peptidoglycan layer with lipid A component

A 44-year-old male patient who has been diagnosed with diabetes mellitus for 6 months visits his primary care physician for a follow-up appointment regarding his blood work. The male insists he has been limiting his sugar intake but has not been consistent with his diabetes medication, Glucophage. Which of the following elevated levels in the patient's blood work reveals that his diabetes has not been effectively controlled? A. Bilirubin B. HbA1c C. Thrombocytes D. HbF E. Erythrocytes

B. HbA1c

A newborn male was admitted to the NICU upon birth due to apparent skeletal deformities and coarse facial features including a depressed nasal bridge and an elongated face. Months later the patient began exhibiting mental disabilities and clouded corneas. Laboratory testing revealed a decreased level of mannose 6-phosphate with elevated levels of lytic enzymes in the patient's plasma. It is determined that the physical deformities are a result of these lytic enzymes causing tissue damage. The patient also has an accumulation of undegraded molecules in his cells. Which of the following best describes this patient's condition? A. Tay-Sachs Disease B. I Cell Disease C. G6PD Deficiency D. Creutzfeldt-Jakob Disease E. PKU

B. I Cell Disease

A 32 year old female presents to her PCP with fatigue, shortness of breath, and lightheadedness. She mentions she recently visited the ED two weeks ago due to a UTI and was treated with a cephalosporin. Upon examining the left upper abdomen, the physician notices moderate splenomegaly. The PCP suspects immunohemolytic anemia. Which of the following antibodies would be the most probable causative agent and under what temperature would it most stably bind to red blood cells? A. IgA; 37 degrees celsius B. IgG; 37 degrees celsius C. IgA; 4 degrees celsius D. IgG; 4 degrees celsius E. IgM; 37 degrees celsius

B. IgG; 37 degrees celsius

A 25 year old female comes into the ER with complaints of fatigue, lethargy, headaches, genital pruritus, and thin, greyish-white discharge. The patient admits to being sexually active and refuses to use contraceptives. The nurse takes the patient's vitals and finds increased heart rate, stroke volume, cardiac output, along with decreased systemic vascular resistance. The doctor orders a CBC test and finds increased erythrocytes, decreased hematocrit, 40% increase in plasma volume, sodium levels are decreased by 4 mEq/L, and urea nitrogen is 50% decreased. A gram-stain of the discharge reveals clue cells that are KOH+. Which of the following is associated with the patient's deficiency? A. Increased nucleotide synthesis B. Increased large, immature RBCs C. Increased PMNs D. Increased Methyl-THF E. Increased Iron concentration

B. Increased large, immature RBCs

Anti-malarial drugs (Chloroquine and Hydroxy-chloroquine) have shown inhibitory effects on other coronavirus in the past (SARVS-CoV-1) and have been proposed as possible treatment for SARVS-CoV-2. One of the mechanisms of action of these drugs is to A. Denature viral RNA B. Inhibit of ACE-2 receptor on host cell C. Inhibit viral protein synthesis D. Activate inflammatory response E. Lyse infected host cell

B. Inhibit of ACE-2 receptor on host cell

A 27 year old woman was making chocolate chip cookies for her mother. After she made them she felt sick and was experiencing watery diarrhea. She goes to the doctor after experiencing these symptoms and she tells him she consumed some of the raw cookie dough which was made with raw eggs. An H2S test revealed a positive result. What is the virulence factor of this pathogen? A. Production of 2 shiga-like toxins (Stx1 & Stx2) encoded on phage B. Invasion of M cells of the small intestine C. Production of heat-labile and heat-stable enterotoxins (LT1 and Sta) D. Production of exotoxins that disrupt the tight junctions in intestinal epithelial cells E. Binding of the toxin to the presynaptic nerve terminal preventing acetylcholine release

B. Invasion of M cells of the small intestine

A patient presents to the urgent care clinic with concern of a persistent ulcer with raised edges that is located on the patient's leg. The patient informs the physician of recent travel to Brazil and recalls being bitten by a "strange looking fly" while hiking. The physician performs a tissue biopsy for identification of the organism. Which organism is most likely to have caused this ulcer? A. Trypanosoma rhodescience B. Leishmania braziliensis C. Trypanosoma cruzi D. Giardia Lamblia E. Clonorchis sinensis

B. Leishmania braziliensis

A 40-year-old male visits his physician and presents with epigastric pain and tenderness as well as vomiting. Examination of concentrated stool samples reveals larval worms that can be distinguished from hookworms by two morphological features. Results of the stool sample confirm infection with Strongyloides stercoralis. This particular nematode burrows into the mucosa of the intestines and releases its larvae into the lumen of the bowel. Which two morphological features would distinguish an infection with S. stercoralis from that of a hookworm? A. Mouthpart location and size of head B. Length of buccal cavity and genital primordium C. Segmentation and distinct shape D. Undulating membrane and nuclei on the tail region E. Larval sheath and microfilariae

B. Length of buccal cavity and genital primordium

Mice infected with SARS/SARS2-RdRp were treated with RDV and showed a dramatic reduction in what? A. Weight loss B. Lung viral load C. Lung hemorrhage D. Pulmonary function

B. Lung viral load

Which COVID-19 protein did not indicate a significant antibody response? A. Nucleocapsid protein (NP) B. Main protease C. S-RBD D. Spike protein E. COVID-19 Histidine tail

B. Main protease

A researcher is studying the different digestive enzymes in the human body. He is focused on one enzyme in particular: ptyalin. To study the efficacy of this enzyme he tests how much of its substrate (starch) gets hydrolyzed in the mouth. Due to the short amount of time food is spent in the mouth, it turns out ptyalin is only able to hydrolyze about 5% of all the starches. This means it is not very efficacious and there must be other enzymes to digest starches later in the digestive system of the human body. What are the product(s) of the breakdown of starch by ptyalin? A. Glucose alone B. Maltose and 3-9 glucose polymers C. Fructose and Glucose D. Maltose alone E. Galactose and Glucose

B. Maltose and 3-9 glucose polymers

A 6-year-old boy is experiencing severe delay in development of motor skills and suffers from hypotonia. Further testing determines the child has an accumulation of undegraded molecules building up in lysosomes. The lysosomes have become nonfunctional and form inclusion bodies because of a lack of which marker on the glycoprotein? A. N-acetylglucosamine B. Mannose-6-phosphate C. N-acetylneuramic acid D. Mannose E. phosphotransferase

B. Mannose-6-phosphate

A 25-year-old male patient presents to the clinic with complaints of fatigue, shortness of breath, and pale skin. Previous medical history reveal that the patient was diagnosed with Alpha Thalassemia. Further lab diagnostics were taken and indicated that the patient has a genetic deletion of two alpha-globin genes from chromosome 16. Based on the deletion of the two alpha-globin genes, what other lab results will be seen in the patient? A. Decreased concentration of Beta-globin chains B. Microcytic Red blood cells and reduced hemoglobin concentration C. Signs for megaloblastic anemia D. Precipitated erythroid precursors due to excess alpha-globin chains E. Deficiency in folate or vitamin B12

B. Microcytic Red blood cells and reduced hemoglobin concentration

A 47-year-old male reports to his PCP for the second time in two weeks after continuing to present with general malaise, headache, chills, and weakness. The man previously reported to his physician after being bit by a tick while on a service trip on Africa. The physician suspected the man had been infected by Plasmodium falciparum after seeing rings form within erythrocytes on a blood smear and prescribed him chloroquine. Since the man still seems to be having symptoms what do you suggest the man needs in terms of treatment? A. Needs a higher dose of chloroquine B. Needs to be prescribed an antiprotozoal C. Needs to be on chloroquine for a longer amount of time D. No medication is needed, and eventually the symptoms will subside E. Needs a blood transfusion

B. Needs to be prescribed an antiprotozoal

A 68 year old man with Chronic Kidney disease, well known to his PCPs office, presents with some of his usual symptoms such as trouble sleeping, reduced exercise tolerance, and some angina. The physician does a blood test and finds decreased levels of iron. What is expected to be the patient's MCV value to be and what protein is causing his diminished iron levels? A. Decreased; hepcidin B. Normal; hepcidin C. Normal; EPO D. Elevated; EPO E. Elevated: hepcidin

B. Normal; hepcidin

A patient presents to the emergency room with abdominal cramps, vomiting, and diarrhea. Most patients with diarrhea should be treated primarily with which of the following types of therapy? A. Intravenous Fluid Therapy B. Oral Fluid Therapy C. Antimicrobial Therapy D. Virotherapy E. All of the above

B. Oral Fluid Therapy

26-year-old female arrived at the ED with complaints of bloody diarrhea, abdominal pain, and fever which onset 2 days ago. In the patient's history, she denied recent travel and reported no changes to her diet but noted that 4 days ago, she was at a neighborhood cookout where she ate grilled chicken, hot dog, and potato salad. Lab results revealed that the patient's symptoms were caused by the bacteria Campylobacter jejuni, most likely from eating undercooked chicken. Which of the following is not a virulence factor of Campylobacter jejuni? A. Motility via flagella B. Pili C. Adherence D. Release of cytotoxin E. Mucosal invasion

B. Pili

A 5 year old male presents to the clinic with persistent abdominal pain and loss of appetite. His mother complains that he has also been feeling nausea and has difficulty swallowing. After performing blood and stool tests, a bacterial infection by Helicobacter Pylori was found. He was given antibacterial therapy to prevent further damage to the gastroduodenal mucosal barrier. Which of the following ways does Helicobacter Pylori lead to a peptic ulcer? A. Failure of the pancreas to secrete pancreatic juice into the small intestine B. Release ammonium that liquifies the barrier and stimulates secretion of hydrochloric acid C. Stomach fails to secrete hydrochloric acid D. Blocks the release of intrinsic factor by parietal cells E.By preventing bowel movements in the large intestine

B. Release ammonium that liquifies the barrier and stimulates secretion of hydrochloric acid

A 23-year-old male presents to the emergency department after training for a Marathon, with complaints of his urine being a red-brown color. Upon further examination it is also noted that the patient presents with signs of Jaundice. It is concluded that this patient is experiencing Intravascular hemolysis, what other clinical manifestations would you expect this patient to experience? A. Splenomegaly B. Renal Hemosiderosis C. Hyposthenuria D. Dactylitis in the hands and feet E. Hemoglobin Barts

B. Renal Hemosiderosis

According to the article what is a promising target for the SARS CoV-2 vaccine? A. Angiotensin Converting Enzyme B. S-RBD C. Nucleocapsid Protein D. Hemagglutinin E. Neuraminidase Protein

B. S-RBD

An 8-year-old male was presented to his pediatrician with complaints of fatigue and shortness of breath. His pediatrician preformed a complete physical exam, including a CBC. His red blood cell count happened to be 5.7 x106 /μL (normal RBC counts for children are 4.0 to 5.5 x106 /μL of blood). It was later found that the child had a mutant erythropoietin (Epo) receptor that is unable to bind SHP-1. Erythropoietin is the hematopoietic cytokine that stimulates production of RBCs. What is true about SHP-1's role regarding RBC production? A. SHP-1 binds to and activates the Epo receptor B. SHP-1 binds to and deactivates the Epo receptor C. SHP-1 causes sustained activation of STAT 5 D. SHP-1 increases RBC production E. SHP-1 phosphorylates and activates JAK2

B. SHP-1 binds to and deactivates the Epo receptor

The parents of a 3 year old girl walk into their primary care physician's office with complaints of watery diarrhea that has worsened to finding blood in the stool. The parents are very concerned as this is their first child. While getting a history, the parents look a bit on edge as they both work full time to keep up with the finances and to allow their daughter to have an amazing future. They state that their daughter spends the day at a private daycare center on the weekdays. After obtaining a physical, it was determined that the illness may have spread via fecal oral from another child attending daycare. Based on this, what is the type of infection that is responsible for causing this patient's watery diarrhea? A. Campylobacter spp. B. Shigellosis C. Salmonellosis D. Yersinia spp. E. ETEC

B. Shigellosis

A 35-year old male was recently diagnosed with a rare hemolytic disease, Paroxysmal Nocturnal Hemoglobinuria (PNH). A unique feature of this disease is the tendency for RBCs to lyse at night/during sleep. What biological feature is responsible for this event occurring? A. Deletion of a single α-globin gene B. Slight decrease in blood pH C. Splicing mutation D. Mycoplasma pneumoniae infection E. G6PD deficiency

B. Slight decrease in blood pH

Which of the following was not one of the criteria used for removing cities/providences from the study? A. Rproxy values being unrealistically high, possibly due to reporting bias B. Some of that data was not significant, meaning the p-value was greater than 0.05. C. Data from providence/city was not available D. Data was available, but could not be properly calculated

B. Some of that data was not significant, meaning the p-value was greater than 0.05.

A young patient presents to a PCP with marked jaundice and is complaining of fatigue, weakness and dyspnea on mild exertion. The physician orders a blood smear, as well as other laboratory tests. The lab tests reveal anemia and the blood smear shows anisocytosis and several dark-appearing spherocytes with no central pallor. Upon further testing and review of the patient's family history the patient is diagnosed with Hereditary Spherocytosis. What medical procedure will be most beneficial for this patient and will help revert the anemia long term? A. Nephrectomy B. Splenectomy C. Partial hepatectomy D. Blood Transfusion E. Subtotal Thyroidectomy

B. Splenectomy

A patient with a prior cardiac history of a prosthetic valve placement comes to the emergency department for weakness and shortness of breath. The emergency department physician orders a CBC test to determine the patient's blood count. The patient is diagnosed with anemia and microscopic inspection of their blood is performed. What is the most likely cause of the patient's anemia and what shape would be present in the patient's RBCs? A. The anemia is most likely caused by a bioprosthetic valve and the RBCs would look normal B. The anemia is most likely caused by a mechanical valve and the RBCs would have schistocytes C. The anemia is most likely caused by a bioprosthetic valve and the RBCs would have schistocytes D. The anemia is most likely caused by hemolytic uremic syndrome (HUS) and would have normal RBCs E. There is not enough information in the question to determine how the patient could be anemic

B. The anemia is most likely caused by a mechanical valve and the RBCs would have schistocytes

A 37 year-old female presents to the emergency department with a 2-day history of fever, nausea, vomiting, and a swollen bug bite on her R ankle. She admits about 4 days ago she was hiking near Lake Tahoe and tried to pet some of the wildlife, mainly squirrels and rabbits. Physical exam was unremarkable apart from the stated bug bite near the patient's ankle with some lymphadenopathy. The attending physician suspects the patient has Yersinia pestis from a flea bite from one of the animals due to an increase in sylvatic plague in the area. A culture of the infected bite proved his suspicions. Which of the following is most likely the bacteria's virulence factor? A. The bacteria's ability to utilize surface adhesion proteins on endothelial cells, attracting macrophages, escaping degradation. B. The bacteria's capsule, it is antiphagocytic and is resistant to serum killing. C. The bacteria's ability to produce pili and adhere to endothelial cells. D. The bacteria's ability to produce toxins LT1 & STa. E. The bacteria's ability to invade M cells and avoid the phagosome and degradation.

B. The bacteria's capsule, it is antiphagocytic and is resistant to serum killing.

A 36-year-old patient presents to his PCP with complaints of a fever, chills, general malaise, vomiting, and headaches. Patient notes that he returned 12 days ago from a visit to sub-Saharan African and the symptoms started 3 days ago. After detecting the Plasmodium species on a Giemsa stain blood smear and confirming the patient has malaria, what chemoprophylaxis does the PCP indicate to take? A. Chloroquine phosphate B. mefloquine C. ivermectin D. metronidazole E. oseltamivir

B. mefloquine

A 25 year old male presents to the clinic with complaints of a penile ulcer. The patient states this occurred 1 month after his vacation in São Paulo, Brazil. He further admits to having unprotected sexual intercourse multiple times during his time in Brazil. Lab results have narrowed down the causative agent to a species in the Enterobacteriaceae family. What is the shape of the causative agent and how will it present on a MacConkey agar ? A. rod, clear B. rod, pink/purple C. rod, green D. cocci, pink/purple E. cocci, cannot grow on MacConkey agar

B. rod, pink/purple

A 30-year-old male patient was brought into the ED by his friends after sweating profusely, having a flushed face appearance, and eventually blacking out at a Superbowl party. Lab results revealed tachycardia (120 bpm), high blood ethanol levels (405 mg/dL), and a low MCV (75 fL/cell). A peripheral blood smear revealed small, hypochromic erythrocytes. The physician would most likely diagnose the patient with what type of microcytic anemia and its effect? A. Iron deficiency; lack of heme synthesis results from the absence of iron incorporating into the porphyrin ring B. Thalassemia minor; globin synthesis disruption C. Acquired Sideroblastic anemia; inhibition of the heme synthetic pathway enzymes D. Congenital Sideroblastic anemia; possible response to pyridoxine and therefore inhibition of the heme synthetic pathway enzymes E. Lead poisoning; blocks iron incorporating into heme

C. Acquired Sideroblastic anemia; inhibition of the heme synthetic pathway enzymes

A college student comes into the ER with complaints of abdominal pain, nausea, vomiting. While taking the student's history, the physician learns that the college student has recently traveled to Cancun, Mexico. The college student explains that his bowel movements have occurred 3 times a day for the past week. What type of diarrhea is the college student experiencing and what is the mechanism responsible for his symptoms, respectively? A. Acute diarrhea, release of neurotoxins B. Persistent diarrhea, attachment to the brush border of intestinal epithelium C. Acute diarrhea, attachment to the brush border of intestinal epithelium D. Persistent diarrhea, release of cytotoxins Persistent diarrhea, release of neurotoxins E. Persistent diarrhea, release of neurotoxins

C. Acute diarrhea, attachment to the brush border of intestinal epithelium

A 60 year old female presents with fatigue, pain in her joints, pale skin, swollen lymph nodes and recent weight loss. When taking the patient's medical history, she informs the physician that she suffers from anemia and takes folic acid occasionally. A Complete Blood Count test was ordered, and the most significant findings were low levels of RBCs and high levels of WBCs. These findings could lead the physician to suspect what possible disease? What additional tests should be ordered to confirm this diagnosis? A. Aplastic anemia; bone marrow aspiration B. Rickets: alkaline phosphatase test C. Acute myeloid leukemia; bone marrow biopsy D. Cutaneous T-Cell lymphoma; skin biopsy E. Multiple myeloma; bone marrow biopsy

C. Acute myeloid leukemia; bone marrow biopsy

A 62-year-old male presents to the ED complaining of fever (101.5 degrees F), abdominal pain, and edema (swelling) to his lower extremities, most notably to the ankles and feet. The patient notes the edema is worse at the end of the day and admits to a history of alcohol abuse. The ER physician suspects liver damage and orders a series of blood tests including a liver function test (LFT). The results of the liver function test were abnormal including low plasma protein levels. What is the main protein constituent of plasma that causes swelling of the ankles due to lowered colloid osmotic pressure? A. Immunoglobulins B. Fibrinogen C. Albumin D. Glucose E. Urea

C. Albumin

A child of Mediterranean descent presents to the clinic with fatigue, facial bone deformities, slowed development and hepatosplenomegaly. After examining lab results along with observed signs and symptoms, the physician diagnosed the child with a type of hereditary anemia. What is the general pathogenesis of this blood disorder? A. Autosomal dominant disorder caused by mutations that affect the red cell membrane skeleton B. Autosomal recessive disorder resulting from a mutation in β-globin that causes deoxygenated hemoglobin to self-associate into long polymers C. Autosomal codominant disorders caused by mutations in α- or β-globin that reduce hemoglobin synthesis D. X-linked disorder caused by mutations that destabilize G6PD, making red cells susceptible to oxidant damage E. Disorder caused by antibodies against normal red cell

C. Autosomal codominant disorders caused by mutations in α- or β-globin that reduce hemoglobin synthesis

A 7-year old girl has gone to the doctor's office with her mother complaining of stomach cramps and regular pale, foul smelling stools. It is also noted that she seems to be growing at a lower rate than her peers at school and has joint pains often. One of the solutions that her doctor gives her is to remove any sort of wheat and rye flour from her diet. Which disease is she most likely to have and where is it most affecting her? A. Celiac disease; Large intestine B. Hirschsprung's disease; Large intestine C. Celiac disease; Small intestine D. Pancreatitis; Small intestine E. Hirschsprung's disease; Small intestine

C. Celiac disease; Small intestine

A 27 year old male had a large cheeseburger for dinner with fries, a large Pepsi, and a slice of pie for dessert. With regard to fat absorption, what is responsible for transporting triglycerides into the thoracic lymph duct? A. Micelles B. Lipase C. Chylomicrons D. Microvilli E. Endoplasmic Reticulum

C. Chylomicrons

A researcher is conducting an experiment on hemopoiesis and has acquired several types of progenitor cells to experiment on. If the researcher wants to begin his work with NK cells, which of the following progenitor cells should he select? A. Megakaryocyte-committed progenitor cells B. Neutrophil progenitors C. Common lymphoid progenitor cells D. Monocyte progenitors E. Eosinophil progenitors

C. Common lymphoid progenitor cells

A 26 year old female enters the emergency clinic with complaints of abdominal cramps, flatulence and foul-smelling watery diarrhea. Upon further questioning it was found that the patient had been on a hiking trip where they attempted to consume untreated water from a mountain stream. Which of the following routes of transmission did the patient most likely acquire this organism? A. Respiratory transmission B. Zoonotic Transmission C. Consumption of untreated contaminated water D. Oral-anal route E. Sexual Transmission

C. Consumption of untreated contaminated water

A 35 y/o male presents to the ED after feeling extremely weak and dizzy for the past 3 days. The patient states that his dizziness has become progressively worse, and has started to feel short of breath. The patient mentions to the physician that he is currently being treated for Non-Hodgkin's lymphoma and is receiving chemotherapy once every two weeks. A CBC test was completed with normal findings other than a low RBC count. Further testing was done showing elevated hepcidin levels. What treatment would be most appropriate to start with for this patient? A. High-dose oral and parenteral cobalamin B. Ferrous sulfate C. Correction of nutritional deficiencies with combined Iron supplementation D. Erythroid stimulating agents (ESA) E. Stem cell transplantation

C. Correction of nutritional deficiencies with combined Iron supplementation

What receptors does MERS-CoV bind to? A. ACE-2 B. human aminopeptidase N (CD13) C. DPP4 or CD26 D. 9-O-acetylated sialic acid

C. DPP4 or CD26

Symptoms of COVID-19 include: A. Increased blood pressure and increased heart rate B. Severe photophobia C. Decreased blood pressure and increased heart rate D. Decreased venous thromboembolism

C. Decreased blood pressure and increased heart rate

A patient has been diagnosed with Addison's disease. Addison's disease is characterized by low levels of aldosterone and cortisol. Which of the following will result in the intestine due to decreased aldosterone levels? A. Increased sodium absorption B. Increased activity of SGLT cotransporter C. Decreased chloride absorption D. Decreased loss of sodium chloride in the feces E. Increased activity of sodium/amino acid cotransporter

C. Decreased chloride absorption

A 30 y/o male presents to ED with abdominal pain, diarrhea, nausea and vomiting. Onset began a week ago and progressively worsened. He notes daily EtOH use with a history of alcohol abuse. On exam patient in severe distress with distended abdomen, severe abdominal tenderness to palpation and jaundice. Labs showed low pancreatic amylase level and he was diagnosed with alcoholic pancreatitis. What significance does low pancreatic amylase have? A. Decreased conversion of Lactose to Galactose B. Decreased conversion of Sucrose to fructose C. Decreased conversion of Starch to Maltose D. Decreased conversion of Maltose to Glucose E. Increased conversion of Starch to Maltose

C. Decreased conversion of Starch to Maltose

A 57 year old caucasian male has been a farmer his whole life. He has noticed over the past week that he has lost weight, is having diarrhea and is experiencing arthralgia. He is seen by his primary care provider and diagnosed with Whipple disease. What morphology did the primary care provider see that led him to this diagnosis? A. Flask shaped ulcers B. Pseudomembranes C. Dense accumulation of distended, foamy macrophages D. Enlarged, sharply delineated Peyer patches E. Abundance of M cells in dome epithelium

C. Dense accumulation of distended, foamy macrophages

A 2-day old newborn develops neonatal jaundice, a condition defined by a yellow tint to the skin and eyes due to elevated erythrocyte destruction leading to high levels of bilirubin. Total bilirubin is the addition of direct and indirect bilirubin measured in the blood. Which of the following is not true regarding direct and indirect bilirubin? A. Conjugation of indirect bilirubin occurs in the liver. B. Indirect bilirubin is bound to albumin in the blood. C. Direct bilirubin is bound to albumin in the blood. D. Indirect bilirubin is more likely to be found deposited in lipid environments. E. Conjugation of bilirubin depends on UDP-glucuronate transferase.

C. Direct bilirubin is bound to albumin in the blood.

Bruce Wayne arrives to the ER with symptoms of a parasitic infection. Unfortunately, the lab had a power outage and tests cannot be run at this time. Because of this the doctor takes an extensive patient history to hopefully narrow down what organisms it may be. Which of the following history questions has the correct organism associated with it? A. Do you own any cats? (Plasmodium falciparum) B. Have you noticed any mosquito bites on you? (Toxoplasma gondii) C. Do you own any cats? (Toxoplasma gondii) D. Have you gone swimming in warm water recently? (Plasmodium falciparum) E. Have you noticed any mosquito bites on you recently? (Naegleria fowleri)

C. Do you own any cats? (Toxoplasma gondii)

A 25 year old medical student is anxiously finishing up final revisions for his GI examination in an hour. The student appears pale, sleep deprived, and slightly diaphoretic. He feels unprepared despite studying hours for the exam. Just a few minutes before the exam starts, the student suddenly has a strong urge to use the restroom and has massive diarrhea. Which of the following could be the most probable cause of the diarrhea? A. Excessive stimulation of the sympathetic nervous system B. Inflammation of the GI tract C. Excessive stimulation of the parasympathetic nervous system D. Immune response from a bacterial infection E. Hyperactivity of parietal cells

C. Excessive stimulation of the parasympathetic nervous system

A 34 year old male presented to his local emergency department for a severe bacterial infection of his lower respiratory tract. To combat the infection, the attending physician started the man on a 14 day intravenous cephalosporin treatment. However, after 10 days of receiving this treatment, he began developing symptoms not typical for pharyngitis. The patient displayed signs of fatigue, severe weakness, shortness of breath and pale, cold skin to the touch. A Direct Coombs Antiglobulin test was then performed which ultimately resulted in significant clumping of the blood. Based on the patient's presentation and the diagnostic test performed, what is the most likely mechanism of action causing the patient's symptoms? A. Intravascular hemolysis due to IgE antibodies binding to the host's red cells at 2°C B. Interrupted synthesis of CD59 regulatory protein due to a mutation in PIGA C. Extravascular hemolysis due to IgG antibodies binding to the host's red cells at 37°C D. Direct hemolysis due to the destruction of the RBC membranes from Heinz Bodies E. A mutation in band 4.2 on the surface of the patient's RBCs

C. Extravascular hemolysis due to IgG antibodies binding to the host's red cells at 37°C

A 68 year old male has an upcoming birthday, and is very excited. However, he has been diagnosed with metabolic syndrome which explains his significantly low HDL levels from last week's bloodwork. Even though he has been advised to watch his diet, he devours cake, ice-cream, and a delicious, greasy birthday dinner. Which of the following best explains the mechanism of fat digestion from his delicious birthday dinner? A. Fat is primarily digested by lingual lipase in the stomach, and only absorbed later in the colon B. Fat emulsification involves polar head groups of bile salts projecting inward in micelle formation, allowing absorption in the duodenum C. Fat is emulsified in the by bile acids, and then digested by pancreatic lipase in the small intestine D. Fat is made more readily fragmentable by lecithin, and absorbed as VLDLs into the enterocyte E. Fat is emulsified in the small intestine by bile acids, and then digested by pancreatic amylase

C. Fat is emulsified in the by bile acids, and then digested by pancreatic lipase in the small intestine

The small intestine is one of the primary sites of nutrient absorption in the human body. Which of the following is not responsible for increasing the mucosal surface area/assisting with absorption? A. Villi B. Microvilli C. Flagella D. Brush Border E. Folds of Kerckring

C. Flagella

What system did the HCQ/CQ meta-analysis use to determine the quality of the data? A. Quality determinant system B. Data efficacy software C. GRADE Profiler Software D. Regression plot

C. GRADE Profiler Software

A 35-year-old female presents to the clinic with unexplained bruising, shortness of breath, fatigue and an abnormal discoloration of her urine close to when she wakes up in the morning. A flow cytometry detects the absence of GPIlinked proteins therefore, diagnosing the paroxysmal nocturnal hemoglobinuria. What makes red blood cells deficient in GPI-linked factors more prone to lysis? A. Damaged bone marrow B. A point mutation in β-globin C. Genetic mutations that inactivate PIGA D. Deficiency in the uptake of vitamin B12 E. Deficient globin synthesis

C. Genetic mutations that inactivate PIGA

A 34-year-old male present to the ED with complaints of fatigue, difficulty breathing, and pallor. His radial pulse is rapid and easily palpable bilaterally. The patient reveals he is on day 6 of 10-day course of penicillin to treat a recent bout of strep throat. CBC results revealed reticulocytosis and peripheral blood smear demonstrates presence of spherocytes. A direct Coombs test confirmed warm hemolytic anemia, suspected to be drug-induced. Through which mechanism did the penicillin induce the hemolytic anemia? A. Folate antagonist mediated B. IgM mediated C. Hapten mediated D. Rh-antibody mediated E. Spectrin mediated

C. Hapten mediated

A wife came home and found her 57 year old husband passed out on the kitchen floor, thinking he was drinking again, she thought nothing of it and left him there. Hours later she sees her husband is still on the floor with pale skin, weakness, and he complains of diplopia. She takes him to the ER where the attending physician orders a CBC, oral examination and physical. The physician finds the patient has glossitis and cheilosis and his red blood cells appear swollen and ovoid. The physician then orders an MRI where neurological demyelination of the dorsal column is evident. Which of the following is most associated with the patient's prognosis? A. Increased rate of purine synthesis B. Increased secretions of intrinsic factor C. Increased bacterial/parasitic growth D. Decreased mean corpuscular volume E. Decreased fat soluble vitamin concentration

C. Increased bacterial/parasitic growth

Which of the following is not a mechanism of action that is targeted by anti-malarial drugs to combat SARS-COV1? A. Inhibition of ACE2 B. Inhibition of viral particle release C. Inhibition of capsule formation D. Anti Inflammatory effect of Interleukin-6 E. Inhibition of pro inflammatory cytokines

C. Inhibition of capsule formation

Carbohydrates are primarily absorbed in the form of monosaccharides, such as glucose. Absorption of glucose occurs in the intestinal tract via a glucose-sodium cotransporter. If sodium wasn't present, what effects, if any, would this have on intestinal glucose absorption? A. Intestinal absorption of glucose will not be affected B. Intestinal absorption of glucose will slow down C. Intestinal absorption of glucose will not occur D. Intestinal absorption of glucose will speed up E. Intestinal absorption of glucose will slow down, adapt, and then return to normal

C. Intestinal absorption of glucose will not occur

A patient presents to the emergency department after having water, noninflammatory diarrhea that now has bright red blood in it. Patient stated that he attended a family picnic 3 days ago with home cooked meals and developed symptoms and developed symptoms yesterday. The patient recalls that he did not see one of their family members wash their hands before serving food. The physician determines that the patient is suffering from acute shigellosis caused by shigella. How does this organism cause bloody stool? A. Destruction of the stomach lining B. Hemolysis of intravascular blood C. Invasion of colonic tissue D. Formation of hemorrhoids E. The patient's bloody diarrhea is unrelated to shigella.

C. Invasion of colonic tissue

A 32 year old female presents to the Emergency Room with a 3 day history of explosive watery diarrhea, with sudden onset on her flight back from Mexico. The physician sends a blood sample to the lab, and 24 hrs later the culture results with growth on MacConkey Agar with lactose fermentation. The oxidase test is negative, and the catalase test is positive. The physician identifies the organism as Escherichia coli. Which virulence factor(s) of the causative organism is responsible for the symptoms exhibited by the 32 year old female? A. P pili B. Invasive plasmid antigen C. LT-1, STa D. Stx1, Stx2 E. Capsule

C. LT-1, STa

A 42 year old patient presents to the emergency room with abdominal pain, cramping, with multiple bloody stools per day and a slight fever. Upon observation, the abdomen is rigid and distended with a suspected elevated diaphragm. A stool sample was taken and trophozoites and cysts were found in the sample, as well as gram negative rods. Which of the following is responsible for removing the trophozoites from the blood? A. Spleen B. Kidneys C. Liver D. Gallbladder E. Diaphragm

C. Liver

A 27 year old woman enters the Emergency Room with a fever, and non-specific complaints of a headache, constipation, and myalgia. The fever has developed over the past few days and is unresponsive to acetaminophen. She has no recent travel history, but she mentions that her mother is visiting from Brazil and arrived 12 days ago. Since arriving, her mom has cooked almost every night. The doctor samples the bacteria and concludes that this is an H2S positive bacteria that colonizes in the gallbladder. What is a trait associated with this bacteria? A. Locus of Enterocyte Effacement Pathogenicity Island B. O157:H7 C. Pathogenicity Island I and II D. Plasmid encoding pINV genes E. Plasmids that encode Fraction 1 gene and Plasminogen Activator Protease gene

C. Pathogenicity Island I and II

A 23-year-old male visits an urgent care after complaints of nausea, vomiting, and diarrhea. Upon physical examination, the physician notes that the patient was scratching their foot, which revealed an irritated rash as well as a non-productive cough. Originally, the patient attributed his rash to ground-bees as they are common for him in his backyard. The physician orders a CBC and OVP. Lab results find that the patient has minor blood loss and non-bile stained segmented eggs are found in his stool. What drug or treatment would be the most beneficial to treat this patient? A. Ivermectin B. Antihistamines C. Mebendazole D. DEC E. Symptomatic

C. Mebendazole

A new breastfeeding mother presents to her OB/GYN concerned that she is not producing breastmilk because of her low lactose intake. Is this a reasonable concern as to why the new mother has a low production of breastmilk? A. Yes, because in order to produce lactose she must intake lactose from some dietary source. B. Yes, because in order to produce lactose the mother must convert lactose to Ca2+ which is the main component of breast milk. C. No, because lactose in the mammary gland is derived from the epimerization of UDP- glucose. D. No, because breastmilk is mainly synthesized from dietary Ca2+, not dietary sugars. E. No, it's not necessary but in order to produce maximal amounts of breastmilk you must and take lactose.

C. No, because lactose in the mammary gland is derived from the epimerization of UDP- glucose.

A 32 year old male with AIDS presents to the clinic complaining of loose, foul-smelling stools, fatigue, and malaise. Careful examination of a concentrated stool sediment and special acid-fast staining reveal the parasite, Cystoisospora belli. What is the diagnostic stage of this parasite that is present in fecal specimens? A. Cysts and large trophozoites containing cilia and macronucleus B. Trophozoites only C. Oocysts D. Cysts and trophozoites E. Small trophozoites with 1 to 2 nuclei and central karyosome consisting of granules

C. Oocysts

A 30-year-old patient went to the hospital because he was not feeling well. The patient had recently traveled to Africa, and was experiencing daily chills, fevers, severe nausea, vomiting, and diarrhea. The doctor ordered thick and thin blood films suspecting a malaria infection. The results showed a high-grade parasitemia consisting of ring forms. Infected RBCs were not enlarged or distorted. Three or even four small rings were seen, as well as appliqué or accolé forms within the individual erythrocytes. What species of Plasmodium was responsible for this patient's symptoms? A. Plasmodium knowlesi B. Plasmodium vivax C. Plasmodium falciparum D. Plasmodium ovale E. Plasmodium malariae

C. Plasmodium falciparum

A Caucasian female of approximately 20 years of age arrives to the ER via ambulance. The EMTs inform the trauma surgeon on call that she was an innocent bystander at a college party where someone pulled a gun, unfortunately for her she took a bullet to the upper leg. She sustained heavy blood loss but fortunately the bullet passed through without impacting any bones or major nerves. Which of the following metrics would you expect to be greater than normal 3 days post-op? A. Intravascular Volume B. Hematocrit C. Reticulocyte Count D. RBC Count E. Platelet Count

C. Reticulocyte Count

A 27-year-old female presents to the emergency room complaining of dysentery, severe stomach pain, and lowgrade fever (99.5 F). She explains that she recently went to the cider mill with her roommate 3 days prior to her symptoms developing. She recalls drinking apple cider and having a few donuts while there. What is the causative agent, and which of the following agars would help differentiate what the causative agent is? A. EIEC, Eosin Methylene Blue agar B. ETEC, Blood agar C. STEC, MacConkey-Sorbitol agar D. STEC, Eosin Methylene Blue agar E. E.ETEC, MacConkey-Sorbitol agar

C. STEC, MacConkey-Sorbitol agar

A 55-year-old male patient initially presented to the clinic with fever, anorexia, nausea, and vomiting. Upon further assessment, small erythematous maculopapular lesions were noticed (Rose spots) on the chest. The patient has a recent travel history to the Philippines, where his friends had him try raw eggs. What is the causative agent responsible for the patient's signs and symptoms? A. Salmonella enteritidis B. Yersinia pestis C. Salmonella Typhi D. Clostridium difficile E. Enterohemorrhagic Escherichia coli (EHEC)

C. Salmonella Typhi

A 19-year-old female presents to her primary care physician complaining of easy bruising on her arms and frequent nosebleeds. Her physician orders a CBC blood test, which shows a low platelet count. Which of the following platelet secretions is responsible for reducing blood flow to areas of blood vessel injury. A. Thromboxane A2 B. λ granules C. Serotonin D. Heparin E. α granules

C. Serotonin

Which of the following were NOT used by the authors to measure immunity responses in patients who recovered from SARS-CoV-2? A. Anti-SARS-CoV-2 IgG antibodies B. Anti-SARS-CoV-2 IgM antibodies C. Serum cytokine profile D. INF-Ɣ secreting T cells E. Cellular immune responses

C. Serum cytokine profile

The parents of a 5-year-old boy walk into their primary care office and are worried about their son. Their child keeps holding onto his stomach, indicating some type of lower abdominal cramps and when he needs to defecate, he is straining to defecate. Parents have reported seeing pus and blood in his stool. While going over his history, his parents state that they work full time through the week and he is mostly at his daycare throughout the day. What type of bacterial infection is presumed to be in this boy based on his symptoms/history? A. Salmonella typhi B. Yersinia pestis C. Shigellosis D. Klebsiella pneumonia E. Proteus mirabilis

C. Shigellosis

After Frodo noticed that his 2 year-old son, Samwise was constantly fatigued, he brought him to their pediatrician. The physician noticed that the toddler had a pallor complexion and after the physician examination, she suspected that the toddler had splenomegaly. She decided to draw blood and ordered a Complete Blood Count with Differential. Later that evening, the Pathologist called her and said that he noticed that the erythrocytes were elliptical in shape. What two proteins are most commonly associated with the shape of the erythrocytes seen? A. DAF & MIRL B. DAF & CD59 C. Spectrin & Protein 4.1 D. Spectrin & Protein 4.2 E. Spectrin & Ankyrin

C. Spectrin & Protein 4.1

A man of Middle Eastern descent is admitted to the emergency room with typhoid fever. The patient is found to have an X linked disorder that results in both intravascular and extravascular hemolysis. Which of the following is not observed in the patient? A. Heinz bodies B. Oxidative stress C. Splenomegaly D. Anemia E. Hemoglobinuria

C. Splenomegaly

A 4 year old boy presents to the ER with abdominal pain and blood in his stools that started yesterday. Mom stated that since it is summer they went to a farm where the child drank fresh milk from a cow. The abdominal pain started 3 days ago and has been getting worse along with some vomiting but no fever. Lab tests showed that the organism that grows colorless on Sorbitol-MacConkey agar. The doctor is worried about complications from this organism. What would be the best treatment for this child? A. Vaccination B. Broad spectrum antibiotic regimen C. Supportive treatment D. Narrow spectrum antibiotic regimen E. Fecal transplant

C. Supportive treatment

A 40-year-old female presents to the hospital for a lung transplant. During her evaluation in pre-op, routine blood tests, as well as serological screenings for blood protozoa were conducted. Among the patient's results, elevated IgG antibodies and low IgM antibodies were seen. During the doctor's evaluation, the patient mentioned that she had 2 indoor/outdoor cats at home and thought that she washed her hands every time that she cleaned their boxes but wasn't 100% sure. If the doctor was to biopsy the patient's original lung tissue after surgery, what would he see that would confirm his diagnosis based on the information mentioned above? A. Leishmania amastigote B. Trypomastigote with an undulating membrane C. Toxoplasma gondii tachyzoites and cysts with bradyzoites D. Multiple ring forms in erythrocytes E.Naegleria trophozoites

C. Toxoplasma gondii tachyzoites and cysts with bradyzoites

An 18-year-old male presents to the urgent care with fatigue, dizziness and a headache. Physical examination noted pale skin, jaundice and splenomegaly. Blood work was ordered, and the results are consistent with hemolytic anemia. Defects in major erythrocyte cytoskeleton proteins are known to lead to hemolytic anemia. Which of the following is NOT a major protein of this cytoskeleton? A. Spectrin B. Actin C. Transferrin D. Ankyrin E. Band 4.1

C. Transferrin

A 6-year-old male arrived at the emergency room accompanied by his parents who stated that he has abdominal pain and distention. Due to irritation and straining during defecation his rectum had prolapsed. This family lives in a remote area with no running water or sanitary sewer. It was stated that they receive food from neighboring unregulated farms. Which is most likely the cause of his condition? A. Dracunculus medinensis B. Enterobius vermicularis C. Trichuris trichiura D. Dirofilaria immitis E. Trinella spiralis

C. Trichuris trichiura

An 18-month-old girl is brought to her PCP. She was recently adopted from South America and brought home with her new parents in the US. Her parents initially brought her in because they noticed she was experiencing chills, fever, and has been severely fatigued, even upon just waking up. Physical exam notes a rash and edema around her eyes. Given her recent history you diagnose this rash as Romaña's Sign and begin to determine your course of treatment. What organism is responsible for this disorder? A. Plasmodium falciparum B. Leishmania donovani C. Trypanosoma cruzi D. Trypanosoma brucei E.Naegleria fowleri

C. Trypanosoma cruzi

A 25-year-old male patient presents to the emergency room with symptoms of fever, headache, and day-time sleepiness. A swelling of the lymph nodes (Winterbottom's sign) is also observed. A review of the patient's history indicates that he recently immigrated to the US from Ghana, West Africa, seventeen days ago. When asked if he remembers anything peculiar that happened before he immigrated, he states that he noticed a bug bite on his arm the day before travelling. He is diagnosed with West African sleeping sickness. Which of the following is most likely the vector through which he got the disease? A. Reduviid bug B. Trypanosoma brucei gambiense C. Tsetse fly D.Trypanosoma brucei rhodiense E. Trypanosoma cruzi

C. Tsetse fly

In central Africa, a 24 year old male visits the office with complaints of fever and pain coming from an ulcer on his leg. The patient looks abnormally hyperactive and has a large bump on the right posterior side of his neck. What is the vector that could've caused these symptoms? A. Sandfly B. Epimastigote C. Tsetse fly D. Trypomastigote E. Promastigote

C. Tsetse fly

The article, "The role of environmental factors on transmission rate of the COVID‐19 outbreak: an initial assessment in two spatial scales," talks about four conducted studies on COVID-19. It also mentions possible reasons where results may vary between each study. Which of the following reasons was not mentioned in the article? A. Using data produced by different public health agencies for the time of evolution and progression of COVID-19, will result in scattered data. B. Using Rt values (effective reproduction number) vs. using dates onset of symptoms will result in scattered data. C. Using White and Pegano method to calculate type-specific reproduction numbers. Thus, shift and complicate the relationship between weather and transmission. D. Using non-pharmaceutical intervention to contain COVID-19 has shown a decreased number of country-wide duration and outdoor transmission.

C. Using White and Pegano method to calculate type-specific reproduction numbers. Thus, shift and complicate the relationship between weather and transmission.

A 25-year-old female patient visits her primary care physician and complains of fatigue, lethargy, and pale yellow tinge on her skin. The doctor orders a CBC and blood smear. Her hemoglobin is 10g/dL (normal is 12-16g/dL). Her RBC count is 3.1x1012L (normal is 4.2-5.4x1012L). The blood smear shows larger than normal RBCs. What is the most likely diagnosis for this patient? A. Microcytic anemia B. Iron deficiency C. Vitamin B12 deficiency D. Sickle cell anemia E. Beta Thalassemia

C. Vitamin B12 deficiency

A 24-year-old female showed up to the emergency room with fever, abdominal cramps, and dysentery. She states that the illness originally started out as watery diarrhea. She was diagnosed with Enteroinvasive E. Coli. What has allowed the bacteria to invade the colonic epithelial cells? A. Actin tails B. Chronic ulceration C. pInv genes D. Shiga toxin E. K1 capsular antigen

C. pInv genes

A 24-year-old female presents to the emergency department with complaints of fever, myalgia, abdominal pain, headache followed by acute watery diarrhea. A routine stool culture showed evidence of inflammation with positive fecal leukocytes and lactoferrin. When asked if she had change anything in her diet, she noted that she recently switched from being fully vegetarian to incorporating lean proteins into her diet such as chicken breast. What is a complication related to the bacteria that she is infected with? A. Hemolytic -uremic syndrome B. Thrombotic thrombocytopenia purpura C. Reiter's Syndrome D. Guillain-Barre Syndrome E. Amebic abscess

D. Guillain-Barre Syndrome

A five year old girl of African descent presents to the emergency department with a low grade fever, cough, and chest pain. An x-ray is performed showing bilateral pulmonary infiltrates. The parents mention that the patient was diagnosed with sickle cell anemia at 8 months old. What factor is most likely increasing the rate of RBC sickling, leading to her symptoms? A. Increased concentration of HbF B. The patient is heterozygotic for the HbS/HbC gene C. Low MCHC D. Decreased transit time of RBC's through microvascular beds E. Increased intracellular pH

D. Decreased transit time of RBC's through microvascular beds

A researcher is studying glucose uptake through the intestinal epithelium. Using animal subjects, he removes the Na/K ATPase transporters on the basolateral side of the epithelial cell membrane. What will happen to the transport of glucose on the apical side in these animals? A. Glucose will enter the cell through an exchange with HCO3- B. Glucose will enter the cell through facilitated diffusion C. Nothing changes D. Glucose will not enter the cell due to the buildup of sodium in the cell E. Glucose will use paracellular diffusion to enter the interstitial fluid

D. Glucose will not enter the cell due to the buildup of sodium in the cell

Glycoproteins are key structural components of the Extracellular matrix. Formation of Glycosaminoglycans (GAGs) requires that a sugar undergo conversion to an amino-sugar and subsequent acetylation to form GAGs such as Hyaluronic Acid. Which of the following is responsible for the addition of an amino group onto Fructose 6-Phosphate to form Glucosamine 6-Phosphate? A. Aspartate B. Serine C. Acetyl Co-A D. Glutamine E. Ammonium

D. Glutamine

A patient presents to the emergency room with fatigue, rapid heart rate, dizziness, and jaundice. After completing numerous lab tests, the physician diagnosis the patient with G6PD Deficiency. The oxidative phase of the pentose phosphate pathway is necessary for the reduction of an important antioxidant called? A. NADPH B. RBCs C. Transketolase D. Glutathione E. NADP+

D. Glutathione

A 19-year-old visits his primary care physician after experiencing problems with his eyes. He wears contacts every day and admits that he does not take them out every night as instructed. His physician takes scrapings from his eye and observes ameba with both trophozoites and cysts present in the tissue. What organism is most likely responsible for his eye infection? A. Naegleria B. Escherichia coli C. Toxoplasma gondii D. Acanthamoeba E.Plasmodium falciparum

D. Acanthamoeba

9. A 20 year old Hispanic man arrives at the Emergency Department with complaints of chest pain, fever, cough, and hypoxemia. The patient explains he is concerned that he has COVID-19 because his symptoms matched the symptoms listed on the COVID-19 checklist on WebMD. The patient is ordered a CT of his chest and is later found to have pulmonary infiltrates in bilateral lobes. Following the patient's chest CT and further lab results, the physician orders an emergency exchange transfusion due to the increasing respiratory compromise. Considering the patient's symptoms and CT findings, which complication of sickle cell disease was the patient diagnosed with? A. Thalassemia Intermedia B. Aplastic Crisis C. α-Thalassemia D. Acute Chest Syndrome E. Cooley's Anemia

D. Acute Chest Syndrome

A 24 year old female presents with the following symptoms: watery diarrhea, abdominal cramps, and vomiting. The lab results from the cultures collected revealed the bacteria was a gram negative rod, lactose fermenter, and growth of pink colonies was observed on MacConkey Agar. These results confirmed that the pathogen was E.coli. Which of the following is the epidemiology of E.coli? A. The most common gram-negative rods isolated from patients with sepsis B. Responsible for causing more than 80% of all community-acquired UTIs C. Prominent cause of gastroenteritis D. All of the above E. A&C

D. All of the above

A 27-year-old male presents to his primary care physician with a pale appearance and describes his symptoms as malaise, general weakness and becoming fatigued rather easily. After a few tests, it was determined that this patient has developed an immunohemolytic anemia. What test did the attending physician use to diagnose the patient's condition based on his desire to characterize the antigens target and temperature dependence? A. A direct coombs antiglobulin test where the patient's serum is tested for its agglutinating ability against commercial red blood cells with specific antigens B. A direct coombs antiglobulin test where the patients red cells were mixed with a serum containing specific human complement activators C. A western blot where the patient's serum was reacted with both a primary and secondary antibody and analyzed on a PDVF membrane D. An indirect coombs antiglobulin test where the patient's serum is tested for its agglutinating ability against commercial red blood cells with specific antigens E. An immunoprecipitation assay where the patient's serum was concentrated and a gene expression analysis was performed to find the autoantibody sequence on his red cells.

D. An indirect coombs antiglobulin test where the patient's serum is tested for its agglutinating ability against commercial red blood cells with specific antigens

A 65 year old man presents to the ED due to chronic fatigue, pale skin, and shortness of breath. The patient had been recently discharged from the hospital a few days prior due to a remission of a malignant tumor. The physician on site suspects a nutritional deficiency and runs a blood test. Results indicate normal WBC and platelet count but a marked reduction in red blood cells. Suspecting anemia, the physician performs a bone marrow biopsy and notes an abnormally high ratio of adipocytes to hematopoietic cells. Which of the following disease conditions below best describes the patient's affliction? A. Hypercellular Bone Marrow B. Pernicious Anemia C. Acute Myelogenous Leukemia D. Aplastic Anemia E. Polycythemia

D. Aplastic Anemia

A 4-year-old child presents to her primary care physician with nausea, vomiting, and diarrhea. Her mother claims she has not seen any blood in the diarrhea. She states that the symptoms started 12 hours ago, after her daughter attended an outdoor birthday party where the children were served chicken fingers. The mother also informs the physician that her daughter is not the only child who attended the party that is now feeling ill. When grown on MacConkey agar, this organism is seen to be lactose non-fermenting. Which of the following is NOT a characteristic of the causative organism? A. Replicates in endocytic vacuoles B. Attaches to the mucosa of the small intestine and invades M cells C. Stimulates cAMP to increase fluid secretion D. Associated with chronic diarrhea and growth retardation in children E. Genes encoded on pathogenicity islands

D. Associated with chronic diarrhea and growth retardation in children

During the check in process at the clinic, a patient responds to the questionnaire by saying he had recently traveled to Eastern Europe. Before walking into the patient's room, the doctor glances at the patient's recent lab work and sees that all of his white blood cell counts are elevated. He remembers there is a new species of helminths that are rampant in Eastern Europe and begins to wonder if that is the cause of the patient's abnormal blood counts. Blood cells involved in protection from parasites, such as worms, are also involved in which of the following activities in the body? A. Degradation of microbial structures B. Smooth muscle contractions C. Initiation of the respiratory burst D. Asthma and allergy responses E. Antibody secretion

D. Asthma and allergy responses

A 64 year old patient living in Saudi Arabia with a history of cardiovascular disease presents to his PCP with fever, cough, rhinorrhea, and change in sense of smell. He mentions he recently went with his family to the beach and reports riding a camel. The causative agent was identified as a positive, single stranded RNA virus. Which of the following could potentially be the receptor the causative agent binds to and what is the potential mortality rate of this patient? A. ACE-2 receptor; 6.3% B. DDP4 receptor; 10.6% C. CD26 receptor: 10.6% D. Both B and C E. All of the above

D. Both B and C

Combined analysis of all 14 patients showed a significant correlation between NAT (neutralizing antibody titers) and number of NP-specific T cells (NP = nucleocapsid protein), which indicates: A. Only T cells participate in the immune response B. Antibodies are not involved in the immune response C. Only NK cells participate in the immune response D. Both T and B cells participate in the immune response E. Cell-mediated response is not involved

D. Both T and B cells participate in the immune response

A 20-year-old female presents to urgent care with complaints of bloating, gas, and abdominal pain. The patient reports that she is lactose intolerant and forgot to take a Lactaid pill prior to consuming ice cream a few hours prior to the visit. Which of the following is the function of the deficient enzyme that is causing the patient's symptoms? A. Breaks down sucrose into glucose and galactose B. Breaks down lactose into fructose and galactose C. Breaks down maltose into two glucose molecules D. Breaks down lactose into glucose and galactose E. Breaks down sucrose into fructose and glucose

D. Breaks down lactose into glucose and galactose

A 45-year-old female visited the ER because she was suffering from a Respiratory Tract infection and the doctor prescribed her to take Ciprofloxacin 500 mg orally every 12 hours to help relieve the symptoms. After a week, the female patient was admitted to the ED with a fever of 103.5 Farheinhiet and complains of abdominal cramps and was dehydrated. The stool sample collected was watery diarrhea with leukocytes presented in the stool but no signs of any blood. The doctor ordered the patient to stop taking the ciprofloxacin and recommend her to take Metronidazole which will be an effective solution. Which of the following pathogenesis would best match the symptoms shown in the patient? A. Enteroinvasive E.coli (EIEC) B. Schistosomiasis C. Enterohemorrhagic E.coli (EHEC) D. C. difficile E. Yersinia

D. C.difficile

A couple present to the clinic after returning from a cruise in the Caribbean. They both complain of fever, chills, vomiting and diarrhea. Due to the recent travel history and adequate time spent in close quarters, the physician believes the two patients have winter vomiting disease. What is the family and genome of this virus? A. Orthomyxovirus, SS -RNA B. Paramyxovirus, SS -RNA C. Rotavirus, DS RNA D. Calicivirus, SS +RNA E. Flavivirus, SS +RNA

D. Calicivirus, SS +RNA

A 12 year old male from Honduras visits the optometrist with complaints of blurred vision, migraines, and ocular pruritus. During the optometrist eye exam, they find ocular larva migrans. They immediately prescribe albendazole to prevent blindness. What is the mode of transmission for the associated diagnosis? A. Walking barefoot B. Ingestion of contaminated water C. Arthropod bite D. Cat feces E. Ingestion of undercooked meat

D. Cat feces

A 33-year-old female was visiting her family in Asia over the summer. She was used to her filtered water here in the states, but her family instructed her that it was safe to drink water from the nearest step well. She came back to the states with no symptoms and carried on with her life. About a year later, she noticed a blister started to form on the side of her foot and went to her doctor immediately. She described a burning sensation and intense pain in her foot where the blister was located. What is the infectious form of the organism described? A. Trophozoites B. Cercariae C. Trypomastigotes D. Copepods E. Promastigotes

D. Copepods

Based on the information presented in the article, it was shown that the use of Tocilizumab in combination with which other drug type, may improve clinical outcomes? A. Antihistamines B. Antivirals C. NSAIDS D. Corticosteroids and Antivirals E. Antibiotics

D. Corticosteroids and Antivirals

A 28-year-old male presents to the emergency department after suffering impalement to the abdomen during a car crash. The patient suffered from massive blood loss and presented to the primary care physician with symptoms of pale skin, weakness, and decreased blood pressure. The primary care physician orders a CBC and Differential and from further questioning and lab results during the patients week long recovery in the hospital, the primary care physician diagnoses the patient with Acute Blood Loss Anemia due to the trauma the patient encountered. What results from the CBC and Differential led to the primary care physicians diagnosis? A. Decrease in platelet production and white blood cell activity B. No effect in platelet production or white cell activity C. Decreased hematocrit and decreased reticulocytes D. Decreased hematocrit and increased reticulocytes E. Decreased hematocrit and increased hemoglobin concentration

D. Decreased hematocrit and increased reticulocytes

A 32 year old male patient visits his primary care doctor for an annual physical check-up. While his doctor was ordering a lab test to check his cholesterol level, she asked him whether he has eaten anything since last evening, to which he responded yes. The patient completely forgot about the fasting instructions and ate a big breakfast this morning. All of the components of his meal, which includes macromolecules of lipid, carbohydrate and protein, are broken down into monomers inside of his body by which of the following chemical reactions: A. Neutralization reaction B. Displacement reaction C. Combustion reaction D. Hydrolysis reaction E. Condensation reaction

D. Hydrolysis reaction

A 70 year old leukemia patient is complaining of peptic ulcer-like symptoms including increased epigastric tenderness, vomiting and diarrhea occurring for the past 3 weeks. An Ova and Parasite Stool and CBC with differential are ordered; results showed elevated eosinophil count and no eggs or cysts detected in the stool. What is the physician's greatest concern regarding the survival of this patient? A. Overgrowth of Candida albicans B. Infection of Entamoeba histolytica C. Infection of Naegleria fowleri D. Hyperinfection syndrome via Strongyloides stercoralis E. Hyperinfection syndrome via Ancylostoma duodenale

D. Hyperinfection syndrome via Strongyloides stercoralis

New parents bring their 1 month old to the ER. Baby has not stopped crying for 24 hours. Patient history shows baby was in the ER two weeks ago for an ear infection. Upon examination white spots on babies' tongues are observed. Laboratory testing reveals the patient has oral thrush. Given the patients history, genetic testing is ordered and shows baby has severe combined immunodeficiency disease (SCID), an X-linked disorder that neither parent presents with. By what mechanism is the babies immune system compromised? A. Decreased production of B-lymphocytes resulting in the lack of JAK3 production B. Erythropoietin receptor that is unable to bind SHP-1 causing sustained activation of JAK2 C. Premature dephosphorylation of STAT dimers D. Inactive B-lymphocytes fail to activate JAK3 due to lack production of T-lymphocytes E. Mutated docking regions preventing the attachment of STATs F. Over-activation of T-lymphocytes causing decreased activation of B-lymphocytes

D. Inactive B-lymphocytes fail to activate JAK3 due to lack production of T-lymphocytes

A 27-year-old female has a blood smear and a complete blood count (CBC) preformed as a part of her routine physical. The patient's vitals are also recorded: BP 120/90, HR 90, and Temperature 98.7°F. The results reveal that the patient has a hemoglobin mutation. This mutation is located at the same position as the HbS hemoglobin mutation. The mutation the patient is diagnosed with is characterized by an amino acid change from Glu to Lys. The patient is clinically unaffected by this hemoglobin mutation. What effect is the hemoglobin mutation having on the patient's hemoglobin concentration within the cells? A. Decrease in water loss from the cell due to a suppression of the potassium transporter resulting in a decrease in concentration of hemoglobin in the cells B. There is no change in hemoglobin levels in the cells because the mutation does not clinically affect the patient C. There is a decrease in solubility of hemoglobin D. Increase in water loss from the cell due to an activation of potassium transporter resulting in an increase in concentration of hemoglobin in the cells E. The patient has sickle cell anemia

D. Increase in water loss from the cell due to an activation of potassium transporter resulting in an increase in concentration of hemoglobin in the cells

A 35-year-old patient went to the hospital this morning because he is not feeling well. The patient appeared pale and often reported weakness, malaise, easy fatigability, dyspnea on mild exertion, and his urine came out dark colored. After several tests the patient was diagnosed with paroxysmal nocturnal hemoglobinuria. This type of anemia can be classified according to its underlying mechanism as one of the following: A. Blood loss: Acute blood loss. B. Increased red cell destruction (hemolysis): Cardiac traumatic hemolysis. C. Decreased red cell production: Infections of red cell progenitors. D. Increased red cell destruction (hemolysis): Deficiency of phosphatidylinositol-linked-glycoproteins. E. Decreased red cell production: Deficiencies affecting DNA synthesis.

D. Increased red cell destruction (hemolysis): Deficiency of phosphatidylinositol-linked-glycoproteins.

17. A 14 year girl comes into the ER with complaints of fatigue and shortness of breath. During the patient's physical examination, the physician finds a rash spread across the patient's back. Upon further testing, it is discovered that the patient has atopic dermatitis, causing extensive tissue damage. What would be the action of the first type of leukocyte to arrive at the site of infection? A. Release of histamine to increase vascular permeability B. Secrete antibodies in response to the antigen-binding occurring at the site of infection C. Activate lysosomes to increase the activity of hydrolytic enzymes and cationic proteins D. Initiate a respiratory burst which creates oxygen radicals to destroy the foreign material E. Engulf microorganisms and necrotic host cell debris

D. Initiate a respiratory burst which creates oxygen radicals to destroy the foreign material

A 24-year-old male presented to his primary care physician with lower neck pain, migraines, and cutaneous nodules on his genitalia. After further examination, the physician diagnosed the patient with primary syphilis. After treating the patient for two weeks with Benzathine penicillin, the nodules turned into painless granulomatous lesions. After further discussing the history of the patient, the physician had learned that the patient recently traveled to Australia on a mission trip 3 months ago. What bacterial infection does this patient have? A. Klebsiella pneumoniae B. Citrobacter koseri C. Klebsiella ozaenae D. Klebsiella granulomatis E. Proteus mirabilis

D. Klebsiella granulomatis

A 2-year old male child presents to the clinic with symptoms of Tay Sach's Disease. However, the observed symptoms appear to be of a progressed stage of the disease. Enzyme assay reveals inactive hexosaminidase A, consistent with the initial diagnosis, but also indicates that hexosaminidase B is inactive in the child. What is responsible for the acceleration of the disease state in this child? A. Mutation of the HexA gene, resulting in an accumulation of ganglioside GM2 B. Mutation of the HexB gene, resulting in an accumulation of ganglioside GM2 C. Mutation of the HexA gene, resulting in an accumulation of ganglioside GM2 and globoside D. Mutation of the HexB gene, resulting in an accumulation of ganglioside GM2 and globoside E. Mutation of the hexosaminidase activator protein

D. Mutation of the HexB gene, resulting in an accumulation of ganglioside GM2 and globoside

A 49 year old male presents to his PCP with complaints of abdominal pain that radiates to his back. He claims that the pain feels worse after he eats, and he usually feels nauseous. When asked about alcohol and drug consumption, the patient claims no prior drug usage but that he has been drinking chronically for nearly 20 years. After performing an amylase test, the serum amylase activity was elevated at 351 units/L (normal range: 30-220 units/L). What is the patient's diagnosis? A. H. pylori infection B. Ulcerative colitis C. Hirschsprung's Disease D. Pancreatitis E. Megaesophagus

D. Pancreatitis

A patient comes to the ER with complaints of pelvic/abdominal pain, odor, and blood when urinating. She explains that she might have a recurring UTI from which she has been suffering from on and off for the past 2 years. After physical examination, the physician decides to obtain a bodily fluid swab. After viewing the swabs contents microscopically, it was confirmed that she had a UTI infection based on the increased levels of a certain cell type that initiates respiratory bursts which create oxygen radicals that rapidly destroy foreign particles. What physical characteristics might have the physician seen under a microscope to confirm his findings? A. Red granules, polymorphonuclear B. Blue granules, polymorphonuclear C. Agranular, blue staining, mono nuclear D. Pink granules, polymorphonuclear E. No nucleus present, granular, cytoplasmic organelles

D. Pink granules, polymorphonuclear

A 5-year-old female child presents to her primary care physician as her parents are concerned about her being unusually fatigued over the past few days. Upon physical examination, it is noted that the child is pale and presents with bilateral icterus. The parents indicate that the child is adopted, and the family history of the birth parents is unknown. Lab evaluation and molecular analysis indicate hemolytic anemia caused by a disorder that is inherited in an autosomal recessive fashion. The disorder results in the mutation of an enzyme that leads to an increase in 2,3-BPG levels in the erythrocyte as a compensating mechanism. A deficiency in which of the following enzymes would explain the child's symptoms? A. Cytochrome b5 Reductase B. Delta - ALA Dehydratase C. Glucose 6-PD D. Pyruvate Kinase E. Ferrochelatase

D. Pyruvate Kinase

An 8-year-old girl is taken to her pediatrician because she is experiencing fluid loss, frequent non bloody diarrhea, and a fever. She had started experiencing these symptoms after receiving brand new livestock and animal feed at her family's farm. The provider knows that the patient is dealing with Salmonella, bacteria that invade through M Cells, but she does not want it to lead to a bacteremia. Which of the following is the correct serotype for this patient, so the provider can treat her correctly? A. Salmonella enterica, serovar. Typhi B. Salmonella enterica, serovar. Infantis C. Salmonella enterica, serovar. Enteritidis D. Salmonella enterica, serovar. Choleraesuis E. Salmonella enterica, serovar. Paratyphi

D. Salmonella enterica, serovar. Choleraesuis

A 30 year old female patient presents to her primary care physician with concerns of her constant watery and bloody diarrhea. Her diet has been normal for her with the exception of eating a rotisserie chicken that she picked up from the grocery store on her way home from work approximately two days ago. A culture has been done and growth on MacConkey agar shows the organism is a non-fermenter and resistant to bile salts. What is the most likely causative organism for the patient's symptoms? A. E. coli B. Staphylococcus aureus C. Bacillus cereus D. Shigella dysenteriae E. Vibrio parahaemolyticus

D. Shigella dysenteriae

Researchers in a lab were comparing blood smear samples from five different patients. After staining each sample with H&E to observe the erythrocytes, they noticed one of the patients blood smear possessed abnormally shaped erythrocytes. Instead of a biconcave disc shape, they found spherical shaped erythrocytes. After further research and tests it was stated that the patient had hereditary spherocytosis. Why would a patient with hereditary spherocytosis be at risk for premature destruction of erythrocytes or hemolysis? A. Spherocytosis results from an autosomal recessive disorder that causes spontaneous hemolysis of the erythrocytes. B. Spherocytosis occurs from an autosomal dominant disorder that causes an extra lipid layer to form around the erythrocyte leading to a spherical shape which triggers hemolysis. C. Spherocytosis increases flexibility of the cytoskeleton of the erythrocyte causing the cell to burst from the increased membrane protein interactions. D. Spherocytosis occurs from defects in anchoring proteins causing the erythrocytes plasma membrane to detach and peel off, this leads to the inability of the erythrocyte to adapt to changes in the environment, ultimately resulting in hemolysis. E. Spherocytosis is a disorder where an increase in erythrocyte integral membrane proteins used for anchoring the plasma membrane to the cytoplasm causes the spherical shape of the erythrocyte, additionally the increase in interactions causes the cell to hemolyse in high pressure environments

D. Spherocytosis occurs from defects in anchoring proteins causing the erythrocytes plasma membrane to detach and peel off, this leads to the inability of the erythrocyte to adapt to changes in the environment, ultimately resulting in hemolysis.

A 50-year-old female presents to the clinic with abdominal pain and diarrhea that has been ongoing for 5 days. Symptoms began after a recent visit to a petting zoo. Her symptoms have progressively worsened with bloody diarrhea. She denies any recent travel or change in diet. Patient with pallor and jaundice on exam. A fecal culture was taken and grown on MacConkey Agar with pink colonies present. What toxin is associated with this pathogen? Lab results: Results Normal range Hemoglobin 9 g/dL 12-16 g/dL Hematocrit 32% 37-43% Platelet 130 x 109 /L 150-400x109 /L Creatinine 1.9mg/dL 0.5-1.1mg/dL A. LT1 and STa B. Enterotoxin C. Hemolysin D. Stx1 and Stx2 E. Plasmid encoded toxin

D. Stx1 and Stx2

A 26 year old male presents to the emergency clinic with complaints of fatigue, pale skin, dizziness, and jaundice. A bilirubin blood test was ordered and shows elevated levels of indirect bilirubin. The patient was diagnosed with hemolytic anemia causing a buildup of unconjugated bilirubin within the blood. Which of the following answers below causes the conjugation and eventual excretion of bilirubin? A. Glucose 6-phosphate dehydrogenase B. Glutathione peroxidase C. Thiamine pyrophosphate D. UDP-glucuronate transferase E. Gluconolactonase

D. UDP-glucuronate transferase

A woman presents to the emergency room with abdominal pain, diarrhea, and vomiting. After inspection of her diarrhea, the physician notes the stool is clear with small flecks of mucus and looks like rice-water. What pathogen is the most likely cause of these stools and how should the patient be treated? A. Campylobacter jejuni, erythromycin B. Campylobacter jejuni, rehydration and electrolytes C. Noncholeraic Vibrio, fluoroquinolone D. Vibrio cholerae, rehydration and electrolytes E. Enterotoxigenic Escherichia coli, fluoroquinolone

D. Vibrio cholerae, rehydration and electrolytes

John Doe presents to the emergency department with complaints of abdominal pain, dysentery, lymphadenopathy, and a temperature of 100.1° F. Last week he was discharged from the hospital after having successfully completed invasive heart surgery during which he received a perioperative blood transfusion. After a physical exam, the physician notes acute lower right quadrant abdominal pain with point tenderness. A stool culture is ordered to confirm a diagnosis of Y. enterocolitica. Which of the following virulence factors is responsible for Yersinia spp. ability to secrete proteins into phagocytic cells that dephosphorylate proteins required for phagocytosis? A. YopJ/P gene product B. YpkA/YopO gene product C. YopE gene product D. YopH gene product E. YopD gene product

D. YopH gene product

A 24 year old patient presents to the ED with complaints of fatigue and pale appearance noticed by coworkers. They have been seen for this issue in the past, and indicates they have been diagnosed with "something that was passed down to me by my parents and makes my blood not right". The patient provides genetic testing results that reveal an autosomal dominant mutation of the membrane proteins spectrin, ankyrin, band 3, and protein 4.2. Upon further examination of the patient's medical history, they have noted splenomegaly seen by radiologists on previous imaging. What should be ordered to confirm this patient's diagnosis and what would be found on this test? A. Blood smear, acanthocytes B. Direct Coombs test, positive agglutination C. blood smear, microspherocytes and elliptocytes D. blood smear, prominent spherocytes E. Indirect Coombs test, positive agglutination

D. blood smear, prominent spherocytes

A Mediterranean patient presents to the emergency room with jaundice and discolored urine. Upon taking his history, it is found that he has recently eaten fava beans. The physician diagnosed the patient with G6PD deficiency. Which of the following occurs when this patient eats fava beans? A. Glutathione concentrations will increase B. Glutathione will become reduced C. Glycolysis will be inhibited D. Oxidative stress will decrease E. Hemoglobin will denature

E. Hemoglobin will denature

A hematologist is preparing a presentation for undergraduate students regarding different cells that are found in the blood that pertain to her research involving evasion of the host immune response. After informing students about the functions of lymphocytes, basophils, eosinophils, which of the following statements is incorrect? A. Lymphocytes are the most common agranulocytes, are not terminally differentiated and have three functionally distinct types: T lymphocytes (involved in cell mediated immunity), B lymphocytes (involved in the production of circulating antibodies) and NK cells (kill virus infected and some types of tumor cells). B. Basophils account for less than 0.5% of total leukocytes, contain Fc receptors for IgE antibodies and CD40L proteins on the plasma membrane surface. The cytoplasm of these cells can have primary granules (variety of lysosomal acid hydrolases) or secondary granules containing heparin, histamine, heparan sulfate, leukotrienes, IL-4 and IL-13. C. Neutrophils are the most common granulocytes and are motile cells and are able to migrate to the area of tissue damage by the expression of adhesion molecules on the neutrophil surface. D. Eosinophils are bilobed cells associated with allergic reactions, parasitic infections and chronic inflammation. Eosinophils have primary granules that contain lysosomal acid hydrolases and secondary granules that contain a crystalloid body made of major basic protein (MBP), eosinophil cationic protein (ECP), eosinophil peroxidase (EPO), and eosinophil-derived neurotoxin (EDN) that can be seen with the TEM. E. Basophils are functionally related to and identical to mast cells of the connective tissue. Both basophils and mast cells originate from basophil-mast cell progenitor (BMCP) cell. Both basophils and mast cells bind to IgE (antibody secreted by plasma cells) binds to Fc receptors after allergen/antigen exposure that triggers the activation of these basophils and mast cells that will go on to release vasoactive agents that are associated with the vascular disturbances associated with hypersensitivity and anaphylaxis.

E. Basophils are functionally related to and identical to mast cells of the connective tissue. Both basophils and mast cells originate from basophil-mast cell progenitor (BMCP) cell. Both basophils and mast cells bind to IgE (antibody secreted by plasma cells) binds to Fc receptors after allergen/antigen exposure that triggers the activation of these basophils and mast cells that will go on to release vasoactive agents that are associated with the vascular disturbances associated with hypersensitivity and anaphylaxis.

A patient presents to the ER with flaccid paralysis. After getting a history from the patient, they admit to consuming an undercooked chicken sandwich. What virulence factor of Campylobacter jejuni is causing the patient's symptoms? A. Spore that passes the blood brain barrier B. Pathogenicity island C. Exotoxin production D. Flagella that invade the CNS E. Cross reactivity of LPS and gangliosides

E. Cross reactivity of LPS and gangliosides

An 8 year old boy is taken to their PCP regarding a fever and dysentery. During the history, the patient does state that he had been to a restaurant 3 days ago and had some poultry that was very pink and looked very unusual; but being a kid, he didn't think about it and ate his dinner. As the provider dwindled down their options, they were coming back to Salmonella. For this Enterobacteriaceae, what is the molecule that induces the mucosal damage, causing dysentery? A. Heat Stable Toxin B. Th1 T Cells C. Type III Secretion System D. Heat Labile Toxin E. Eicosanoid Hepoxilin A3

E. Eicosanoid Hepoxilin A3

A 23 year old patient comes into the ER with non bloody diarrhea. The morphology of the bacteria is a "stacked brick" appearance. What strain of Enterobacteriaceae does the patient have? A. Enterohemorrhagic E. coli B. Enterotoxigenic E. coli C. Enteropathogenic E. coli D. Enteroinvasive E. coli E. Enteroaggregative E. coli

E. Enteroaggregative E. coli

Upon being picked up from daycare, the parents of a 4 year-old male noticed the child showing exacerbated agitation around his perianal region, as displayed by excessive itching and irritation. Upon examination by his pediatrician, the child's diagnosis was confirmed with the tape test. What family of parasitic helminths does the causative organism belong to? A. Trypanasoma Cruzi B. Ancylostoma duodenale C. Trichuris Trichiura D. Onchocerca volvulus E. Enterobiasis

E. Enterobiasis

A 6 month-old male presents to the clinic because his parents have noticed he has been having difficulty eating, crawling and remaining in an upright position. The physician decided to run a serum test to measure enzymatic activity. The results showed no hexosaminidase A and hexosaminidase B activity. What type of glycolipid(s) are expected to be accumulated in lysosomes? A. GM2 ganglioside B. Globoside C. GM1 ganglioside D. Ceramide E. GM2 ganglioside and Globoside

E. GM2 ganglioside and Globoside

A 42-year-old female presents to the emergency department with abdominal pain, nausea, vomiting, and heartburn. She states that antacids seem to help a little, but don't completely take away the symptoms. In taking a medical history, the physician discovers that the patient has been taking a large amount of NSAIDs for months due to chronic knee pain. The physician suspects a peptic ulcer. Which of the following is not a cause for a peptic ulcer? A. Irritation B. High acid and pepsin content C. Poor blood supply D. Infection with H. pylori E. Increased pyloric gland secretions

E. Increased pyloric gland secretions

A 71 year old man presents to the ER with diarrhea and intense pain in the joints. He notes symptoms began while harvesting strawberries and herding cattle as his primary occupation is farming. He also suspects to have lost weight recently and reports an increased tendency to experience food cravings. After ordering a CBC, the doctor notes an increase in macrophages. Physical exam is not notable with exception of pain during lymph node examination. Which of the following answers is most likely associated with the patient's condition? A. Increased lymph drainage B. Upregulation of macrophage synthesis in lymph nodes C. Accumulation of granulocytes D. Increased absorption in small intestine E. Lymphatic obstruction

E. Lymphatic obstruction

A 27-year-old male presents to his PCP with complaints of exhaustion, dizziness, episodes of anxiety, and dark colored urine. He explains that his symptoms started 4 days ago but have been getting progressively worse. When his doctor asks for any changes in diet, the man explains that he and his friends have gone out to dinner twice in the past week to a Mediterranean restaurant where they shared a family sized chicken kabob pita, lentil soup, fava bean appetizer, and baklava. The doctor orders a blood smear and notes the presence of bite cells among normal red blood cells. What is the cause of the red blood cell abnormality and where does it occur? A. Decreased level of reduced glutathione causing a deformation of the membrane of a red blood cell in circulation B. Misfolded beta globin protein during formation in bone marrow C. Cross-linking of reactive sulfhydryl groups on globin chains while passing through pulmonary capillaries due to high levels of oxidants D. Hemolysis of RBCs in circulation via leukocytes E. Macrophages plucking Heinz bodies from red blood cells as they pass through the splenic cords

E. Macrophages plucking Heinz bodies from red blood cells as they pass through the splenic cords

A medical student presents to the primary care physician with complaints of general malaise and fatigue. She reports that yesterday she had been out drinking when she accidentally walked into a closed glass screen door and got a bloody nose that lasted about 30 minutes. She was celebrating because she had just finished taking an exam that she had studied extremely hard and had stressed over for about a week. The physician orders blood work and determines that the patient is suffering from chronic blood loss. What is most likely the cause of the blood loss and what test would the physician use to confirm the diagnosis? A. Nosebleed and peripheral blood smear B. Peptic ulcer and peripheral blood smear C. Nosebleed and stool sample D. Sickle cell anemia and peripheral blood smear E. Peptic ulcer and stool sample

E. Peptic ulcer and stool sample

Last week, Michelle went out to Jack in the Box with her friends. Three days later, she began having watery diarrhea, which progressed to bloody diarrhea. Michelle is a naturally unlucky human being, but still decided to do absolutely nothing about her symptoms. Michelle starts feeling extreme fatigue and notices blood in her urine. Finally, she goes to the doctor where she is told that her EHEC infection has progressed to Hemolytic Uremic Syndrome (HUS). She is placed on supportive care. After the doctor examines Michelle's peripheral blood smear, she diagnoses her with Microangiopathic hemolytic anemia (MAHA). What did the doctor find on the peripheral blood smear that led her to this final diagnosis? A. Spherocytes B. Teardrop cells C. Pencil-shaped deformities D. Hypersegmented neutrophils E. Schistocytes

E. Schistocytes

A 6 year old presents to the emergency room with her mother. The mother states her daughter started having watery diarrhea yesterday in addition to abdominal cramping and a fever. Both the mother and the daughter have never traveled outside of the U.S. and the daughter attends a day-care center after school. She is diagnosed with shigellosis. Based on the travel history, after school activities, and symptoms, which species of shigella is most likely causing the daughters symptoms? A. Shigella flexneri B. Shigella dysenteriae C. Shiga-toxin E. coli D. Shigella boydii E. Shigella sonnei

E. Shigella sonnei

A 17-year-old male arrives to the ER following a large car accident on the freeway. During the accident a piece of metal sliced into the patient's thigh, cutting their femoral artery in the process and causing a massive amount of blood loss. Which of the following changes are not expected to occur during the patient's bodily response to the blood loss? A. The patient will develop leukocytosis B. The patient's blood volume will initially be restored by water from the ISF C. The patient will have an increased reticulocyte count D. The patient's hematocrit will be lower than the average value E. The patient's blood volume will initially be restored by a rapid increase of bone marrow activity

E. The patient's blood volume will initially be restored by a rapid increase of bone marrow activity

A 37-year-old female presents to the clinic complaining of nausea, abdominal pain, and diarrhea. During the patient interview, she states the symptoms did not begin until after she moved back to her childhood home to help her aging parents run their cattle farm. Due to the general nature of her symptoms, a stool sample is taken for examination. Microscopy shows the presence of sporocysts in her stool. What is the method of treatment for this patient? A. Metronidazole, followed by iodoquinol B. Doxycycline C. Trimethoprim-sulfamethoxazole D. Pyrimethamine and Sulfadiazine combination E. There is no known treatment method

E. There is no known treatment method

A researcher in a clinical lab is investigating the potential of the CRISPR system for possible gene therapeutics. In efforts to test the effectiveness of CRISPER/Ca9, the researcher decides to knockout the gene responsible for the transaldolase enzyme in the Hexose Monophosphate Shunt and monitor levels of Fructose 6-phosphate production. To the researchers' surprise, Fructose 6-phosphate was still produced. What enzyme in the Pentose Phosphate Pathway may be responsible for the production of Fructose 6-phosphate? A. Gluconolactonase B. Hexokinase C. Glucose 6-phosphate dehydrogenase D. Epimerase E. Transketolase F. Glucokinase

E. Transkelolase

A 35-year-old female patient visits the Emergency Department 3 months after returning from a Missionary trip to Haiti. She complains of abdominal pain with light distention and weakness. The physician also notices that the vitals recorded by the technician indicate moderate weight loss, although the patient claims she believed it to be the result of her trip. Following a stool examination, the results reveal bile-stained eggs with polar plugs. Considering the patient's recent travel history and laboratory analysis, what parasitic organism is most likely causing the presented symptoms, and what final location do the adult worms migrate to? A. Ascaris lumbricoides; cecum B. Trichuris trichiura; hepatic portal C. Necator americanus; skin D. Ancylostoma duodenale; anus E. Trichuris trichiura; intestinal mucosa

E. Trichuris trichiura; intestinal mucosa

A 27-year-old male is brought into the Urgent Care Clinic by his parents with complaints of muscle pain, fever, and rigors. His parents inform you that he has been "acting strange" and has unintentionally lost a lot of weight within the last week. Upon obtaining a travel history they explain how they returned the previous week from visiting their family in Africa. The physician suspects the patient has a parasitic infection by T. b. rhodesiense which is confirmed by blood and spinal fluid examination. Which of the following is not a characteristic of T. b. rhodesiense? A. Myocarditis B. Kidney damage C. Rapid CNS disease D. Rapidly fatal E. Winterbottom sign

E. Winterbottom sign

Bilirubin indications:

This test is used to evaluate liver function - it is part of the evaluation of adult patients with hemolytic anemias and newborns with jaundice. components: total bilirubin, direct (conjugated) bilirubin, indirect (unconjugated) bilirubin (indirect bilirubin makes up 70 - 85% of total bilirubin) (low -n/a)

A scientific study was conducted and summarized in the article, "The role of environmental factors on transmission rate of the COVID‐19 outbreak: an initial assessment in two spatial scales." The group of scientists was led by which of the following scientists? a. Poirier Et. al (2020) b. Dickerson Et. al (2019) c. Schmeikel Et. al (2020) d. Thomas Et. al (2019) e. Liu Et. al (2020)

a. Poirier Et. al (2020)

The article, "The role of environmental factors on transmission rate of the COVID‐19 outbreak: an initial assessment in two spatial scales," talks about different potential causes of COVID-19 outbreak and increased transmission rate. Which of the following potential variables were discussed in the study? a. Warm weather with high humidity b. Warm weather with low humidity c. Cold weather with low humidity d. Cold weather with high humidity

a. Warm weather with high humidity

A 25 year old patient complains about a high fever, chills and nausea. He mentions that he had recently traveled to Malaysia. A microscopic test is performed and he shows high levels of parasitemia in his red blood cells. The organism found has multiple parasites in the red blood cells and has a distinct 24 hour life cycle. Which is the most likely parasite that is causing the patient's symptoms? a. Plasmodium Falciparum b. Plasmodium Knowlesi c. Plasmodium Malariae d. Plasmodium Vivax e. Toxoplasmosis Gondii

b. Plasmodium Knowlesi

Nancy C., female, 43-year-old, arrived to the emergency room with complaints of an extremely painful nodule on her left ankle. Upon examination, the attending physician determined the nodule to be a pruritic Calabar swelling. The last time Nancy traveled out of the country was 14 months ago, in which she was exploring the congo river basin on her honeymoon. What is the likely causative agent and the vector? A. ​L.loa and blackflies B. L.loa and monkeys C. L. loa and Chrysops, Mango fly D. Wucheria bancrofti and mosquito E. Erichinella and rabid ferrets

c) L. loa and Chrysops, Mango fly

COVID-19 is caused by a coronavirus called SARS-CoV-2. Older adults and people seem to be at higher risk for developing more serious complications from COVID-19 illness. Where did COVID-19 originate from? a. Guangdong Province b. Hubei, China c. Rome, Italy d. Wuhan, China

d. Wuhan, China

LDL indications

low-density lipoprotein, bad cholesterol; carries cholesterol from the liver to cells in the body


Kaugnay na mga set ng pag-aaral

Intermediate Accounting Chapter 13, 14 & 15. Current Liabilities and contingencies. Long-term Liabilities. Stockholders' equity.

View Set

Vocabulary Workshop Level H Unit 4 Completing the Sentence

View Set

Chapter 25/26 Quiz study information

View Set